SlideShare a Scribd company logo
1 of 124
Gastroenterology

UNSOM
Emergency Medicine
Review

1/16/2007

UNSOM: EMR
Dysphagia (1)
•
•
•
•

Difficulty swallowing
Solids: mechanical/obstructive
Solids/liquids: motility disorder
Oropharyngeal dysphagia (transfer): neuromuscular disorder
(CVA)
• Progressive (CA) vs. non-progressive (web)
• Strictures 2° reflux (can mimic CA)
• Work up
 Esophagram
 Endoscopy
 Esophageal motility studies

1/16/2007

UNSOM: EMR
Dysphagia (2)
• Infectious: Botulism, diptheria, polio, rabies,
Sydenham’s chorea (rheumatic fever), tetanus
• Immunologic: Scleroderma, multiple sclerosis,
myasthenia, ALS, polymyositis, amyloidosis
• Motor dysfunction:
CN palsy (posterior CVA), diabetic neuropathy
Achalasia (vomit undigested food)
Aperistalsis of esophagus (loss of
Auerbach’s plexus in the esophagus)
Tx: CCB (Diltiazem, nifedipine)
–Botox, dilation, myomotomy
1/16/2007

UNSOM: EMR
• A 32-year-old woman presents with chest pain that has
worsened over the past 2 months. She says it gets worse
when she lies flat or exercises and after she eats or drinks
quickly. She has no significant past medical history, but
her husband says she has lost about 10 pounds recently
and has been throwing up undigested food. What are the
expected diagnostic findings?
• A. Diffuse ST-segment elevation and PR-interval
depression
• B. Dilated esophagus proximal to a beaklike lower
esophageal sphincter
• C. Gastric inflammatory changes
• D. White matter plaques in the brainstem
1/16/2007

UNSOM: EMR
• A 32-year-old woman presents with chest pain that has
worsened over the past 2 months. She says it gets worse
when she lies flat or exercises and after she eats or drinks
quickly. She has no significant past medical history, but
her husband says she has lost about 10 pounds recently
and has been throwing up undigested food. What are the
expected diagnostic findings?
• A. Diffuse ST-segment elevation and PR-interval
depression
• B. Dilated esophagus proximal to a beaklike lower
esophageal sphincter
• C. Gastric inflammatory changes
• D. White matter plaques in the brainstem
1/16/2007

UNSOM: EMR
Dysphagia (3)
Mechanical
• Zenker’s diverticulum
Pharyngoesophageal pouch
Proximal: above the UES
Elderly, regurgitation of undigested food

• Cancer: MCC = squamous
Risk factors: smoking, achalasia, caustic ingestion

• Extraluminal obstruction / tumor

1/16/2007

UNSOM: EMR
Dysphagia (4)
Mechanical

• Strictures
GERD, chronic inflammation, occur in distal
esophagus
• Schatzki’s ring
Fibrous structure distal esophagus
MCC of intermittent dysphagia, steakhouse
syndrome
• Webs (occurs intermittently)
Circumferential mucosal outpouchings
Congenital or acquired
Plummer - Vinson Syndrome = symptomatic
hypopharyngeal webs + iron deficiency anemia
1/16/2007

UNSOM: EMR
Odynophagia
• Odynophagia - pain upon swallowing
spasm - painful muscle contraction
• Causes of esophagitis
Reflux
Infection: candida, herpes, immunosuppression:
(HIV, DM, steroid use, CA)
Inflammatory conditions: infection, radiation,
trauma, foreign body
• Admit dysphagia, odynophagia
Bleeding
Rupture
Severe dehydration
Malnutrition
1/16/2007

UNSOM: EMR
• Which of the following patients requires oral fluconazole
treatment?
• A. 17-year-old girl with both dysphagia and odynophagia
refractory to acid suppression therapy who also has
multiple allergies
• B. 27-year-old man with chest pain and severe
odynophagia who also has asthma and is HIV positive
• C. 47-year-old man with transport dysphagia for solids
initially and now liquids who also smokes
• D. 55-year-old man with halitosis, transfer dysphagia, and
neck fullness

1/16/2007

UNSOM: EMR
• Which of the following patients requires oral fluconazole
treatment?
• A. 17-year-old girl with both dysphagia and odynophagia
refractory to acid suppression therapy who also has
multiple allergies
• B. 27-year-old man with chest pain and severe
odynophagia who also has asthma and is HIV positive
• C. 47-year-old man with transport dysphagia for solids
initially and now liquids who also smokes
• D. 55-year-old man with halitosis, transfer dysphagia, and
neck fullness

1/16/2007

UNSOM: EMR
Hiccups (Singultus)
• Involuntary stimulation of the respiratory reflex
with spastic contraction of inspiratory muscles
on closed glottis
• Benign causes: gastric distention, smoking,
ETOH, change is environmental temperature
• Persistent: damage to vagus/phrenic
nerve/CNS
 Continue with sleep: organic
 Relieved with sleep: psychogenic

• Organic
 CNS: neoplasm, MS, ICP
 PUD, tonsillitis, goiter, pericarditis, pacemaker, STEMI
1/16/2007

UNSOM: EMR
Esophageal Rupture (1)
• MCC iatrogenic
#1: Endoscopy
#2: Dilation
MCC in ED: NG tube
Diagnosis by esophagram

• Mallory - Weiss - partial thickness tear
Location: GE junction
5-15 % of UGI bleeds
Vomiting, retching
Risk factors: ETOH, hiatal hernia
Spontaneous resolution common
1/16/2007

UNSOM: EMR
Esophageal Rupture (2)
• Boerhaave’s Syndrome - full thickness tear
Males usually, age 40-60
Typically associated with alcohol (50%)
Typically left posterior distal rupture
Chemical, then infectious mediastinitis
Severe chest pain, shock, sepsis
Air in mediastinum (Hamman’s crunch)
Pyopneumothorax
Gastrografin (water soluble) UGI
Fluids, Antibiotics, Surgical consult

• X-ray: mediastinal air, left pleural effusion,
pneumothorax, widened mediastinum
1/16/2007

UNSOM: EMR
Pneumomediastinum / Subcutaneous
Emphysema

1/16/2007

UNSOM: EMR
Esophageal Foreign Bodies
• Levels of narrowing
MCC: Cricopharyngeus muscle (C6) (<4 y/o)
Aortic arch (T4)
Tracheal bifurcation (T6)
Gastroesophageal junction (least) (T11)

• Coin x-rays
AP orientation = trachea (same plane as vocal
cord orientation)
Transverse orientation = esophagus
1/16/2007

UNSOM: EMR
Foreign Body

1/16/2007

3

UNSOM: EMR
Esophageal Foreign Body

1/16/2007

UNSOM: EMR
Esophageal Foreign Bodies (3)
• 10-20% require some intervention
• 1% demand surgical treatment
• Most foreign bodies will pass if they traverse
the pylorus
• Soft drink pull tabs - may not show up on x-ray

1/16/2007

UNSOM: EMR
Esophageal Foreign Bodies (4)
• Indications for endoscopy
Sharp / elongated
Button batteries
Perforation
Nickel / quarter at C6 (pediatric)
In esophagus > 24 hours

1/16/2007

UNSOM: EMR
Esophageal Foreign Bodies (5)
• Button batteries
 Double density radiographically
 Must always be removed from esophagus immediately
 Rapid burns with perforation < 6 hours (Lithium worse)
 Batteries do not need to be removed:
Passed esophagus, asymptomatic
Passed the pylorus <48 hours
 Most will pass completely in 48-72 hours, serial radiography
• Treatment: broad-spectrum ABX, surgical consultation

1/16/2007

UNSOM: EMR
Foreign Bodies (6)
Sharp objects
> 5cm long & 2cm wide
Magnet + metal
All others: serial exam / x-rays
Fish/Chicken bones or plastic  CT

1/16/2007

UNSOM: EMR
Sharp Foreign Body
Sharp Foreign Body

1/16/2007

UNSOM: EMR
Large-corrosive-impacted
Foreign Body

1/16/2007

UNSOM: EMR
Esophageal Food Impaction
• Most patients with food impaction have
underlying esophageal pathology
• Must evaluate for cause after dislodgement
• Treatment options:
Glucagon - relaxes distal esophageal sphincter
Nifedipine - reduces lower esophageal tone
Carbonated beverages - gaseous distention
may push the bolus into the stomach
Endoscopy
No papain (meat tenderizer)
1/16/2007

UNSOM: EMR
Caustic Ingestions (1)
• Acids (+/- bad)
Coagulation necrosis
No ongoing tissue necrosis

• Alkali (bad)
Liquefaction necrosis (pH 12.5)
Ongoing tissue necrosis

• Severity
Nature, volume and concentration
Tissue contact time
Presence or absence of stomach contents
Tonicity of pyloric sphincter
1/16/2007

5
UNSOM: EMR
Caustic Ingestions (2)
• Inconsistent relationship between oral signs /
symptoms and esophageal findings
• All patients with 2-3° burns are symptomatic
• Diluents - water / milk only for solid alkali
• No neutralizers = exothermic generation of heat

Endoscopy best diagnostic tool
• Complications
Early: acute airway compromise due to edema,
perforation
Late: stricture, perforation
1/16/2007

UNSOM: EMR
Peptic Ulcer Disease
• Incidence decreasing in general population and
increasing in the elderly (liberal use of NSAIDs)
• MCC Duodenal (80%), gastric (20%)

• Helicobacter pylori responsible for most
• Predisposing factors:
• Treatment:

- antibiotics against H. pylori (amox, clarithro, metro)
- histamine blockers (histamines stimulate acid inhibitors)
- parietal cell inhibitors (omeprazole)
- ulcer surface protectants (sucralfate)

• Complications:
1/16/2007

- smoking, alcohol
- type O blood
- NSAIDs and steroids

- bleeding
- perforation (can cause pancreatitis)
(do upright CXR for free air)
- obstruction
UNSOM: EMR
Bilirubin (1)

• Breakdown product of hemoglobin
• Hyperbilirubinemia
• Unconjugated (> 85%) (“indirect reacting”)
Increased bilirubin load (hemolysis)
Inability to conjugate (Gilbert’s, neonatal, sepsis)

• Conjugated (< 30%) (“direct reacting”)
Decreased ability to excrete from biliary
tree = cholestasis / obstruction
Intrahepatic cholestasis
Hepatocellular damage
Damage to biliary endothelium

Extrahepatic cholestasis
1/16/2007

Biliary outflow obstruction (stones, mass,
congenital inflammation, CHF)

UNSOM: EMR
Bilirubin (2)
• Conjugated bilirubin in bowel is converted by
gut bacteria to urobilinogen

• Urobilinogen is absorbed from the gut into the
circulation and excreted in urine
• If jaundice is present but urine urobilinogen is
negative = excess unconjugated
hyperbilirubinemia

• If jaundice is present but excess positive urine
urobilinogen = excess conjugated bilirubin
1/16/2007

UNSOM: EMR
Hepatitis (1)
• Causes - viral and toxic
Malaise, jaundice, increased SGOT, increased
bilirubin
Alcoholic hepatitis
Abnormal protime is a marker indicating
significant liver dysfunction
if elevated,
consider altering or holding doses of livermetabolized drugs

• Viral Type A

1/16/2007

Fecal - oral, onset 2 weeks post-exposure
Prophylaxis - immune globulin within 2 weeks of
exposure (travelers, household contacts)

UNSOM: EMR
Hepatitis (2)
• Viral Type B
Percutaneous, parenteral or sexual exposure
Onset 1-6 mo (mean = 75 days) post-exposure
Complications = cirrhosis, liver cancer, carrier
state (10%)

• Markers
HBsAg:
HBsAb:
HBcAb:
HBeAg:

+ early (before enzymes increase) Infective
+ 2-6 mo after clearance of HBsAg Immune
+ 2 wks after + HBsAg * persists for life
+ implies high infectivity

*May be the only positive marker during the window
when HBsAg declining and HBsAb increasing
1/16/2007

UNSOM: EMR
Hepatitis (3)
• Hepatitis B exposure - source known HBsAg
positive
• Unvaccinated
HBIG ASAP + vaccination (0, 1 mo, 6 mo)

• Vaccinated
Incomplete series- vaccine booster
Known responder- test for HBsAb if > 10,
no rx; if < 10 HBIG and vaccine booster
Known non - responder - HBIG x 2 (0, 30 days)

1/16/2007

UNSOM: EMR
Hepatitis (4)
• Hepatitis B exposure - source unknown

• Unvaccinated
Initiate vaccination

• Vaccinated
Same as for HBsAg positive source

• HBIG only recommended if source or situation
maybe high risk for exposure
1/16/2007

UNSOM: EMR
Hepatitis (5)
• Viral Type C
Percutaneous, parenteral or sexual exposure
Usual cause of non-A, non-B hepatitis
High carrier rate, higher incidence in HIV
Cirrhosis / liver cancer (50%)
2% seroconversion

• Indications for hospitalization (any hepatitis)
Encephalopathy, PT/INR significantly increased,
dehydration, hypoglycemia, bilirubin over 20, age
over 45, immunosuppression, diagnosis uncertain
1/16/2007

UNSOM: EMR
Hepatic Encephalopathy
• Precipitants = “LIVER” (Librium [sedatives],
Infection, Volume loss, Electrolytes disorders, Red
blood cells in the gut [a major cause])
• Others: dietary protein excess, worsening
hepatocellular function
• Early sign = “sleep inversion” - sleeping during the
day / awake at night
• Asterixis (“liver flap”)
• Ammonia levels: arterial more helpful than venous
• Check for hypoglycemia!!!
• Treatment: Oral or rectal neomycin / lactulose /
decrease dietary protein / avoid sedatives / avoid
bicarbonate (alkalosis can worsen encephalopathy)

1/16/2007

UNSOM: EMR
Spontaneous Bacterial Peritonitis
• Occurs with chronic liver disease
Portal hypertension
bowel edema
migration and leakage of enteric organisms (E.
coli 50%, enterococcus 25%)
• Abdominal tenderness, worsening ascites,
encephalopathy, fever, sepsis, shock
• Diagnosis: paracentesis with increased WBC
PMN >250/ul
• Tx: Ceftriaxone, ppx: Cipro or Bactrim
1/16/2007

UNSOM: EMR
• A 57-year-old man with a history of cirrhosis
presents with acute renal failure. He denies
recent illness and is not taking any nephrotoxic
medications. He is well hydrated; his urinalysis
is negative. Which of the following is the
definitive treatment?
• A. Hydration
• B. Liver transplant
• C. Renal transplant
• D. Transjugular intrahepatic portosystemic
shunt
1/16/2007

UNSOM: EMR
• A 57-year-old man with a history of cirrhosis
presents with acute renal failure. He denies
recent illness and is not taking any nephrotoxic
medications. He is well hydrated; his urinalysis
is negative. Which of the following is the
definitive treatment?
• A. Hydration
• B. Liver transplant
• C. Renal transplant
• D. Transjugular intrahepatic portosystemic
shunt
1/16/2007

UNSOM: EMR
Gallbladder (1)

• Stones = mostly bilirubin / cholesterol (radiolucent)
• Biliary colic = pain, vomiting, due to obstruction
by stones without inflammation
• Cholecystitis (stone-related = calculous)
MCC of abdominal pain in the elderly
OR
Obstruction
distention pain / vomiting /
inflammation
infection (usually E. coli,
Klebsiella)
increased WBCs
• Rupture of stone into small bowel with obstruction at
ileocecal valve = GALLSTONE ILEUS
Air in biliary tree (from bowel) = pneumobilia

1/16/2007

UNSOM: EMR
Gallbladder (2)
• Acalculous cholecystitis
No stones
5-10% of cases
Usually a complication of another process
(trauma, burn, postpartum, postop, narcotics)
Patients often quite sick
Likely cause of GB perforation
Increased risk with diabetes and elderly
Greater morbidity than calculous variety

• Ascending cholangitis
Infection spreading through biliary tree
Charcot’s triad = jaundice, fever, RUQ pain
1/16/2007

UNSOM: EMR
1/16/2007

UNSOM: EMR
Gallbladder (3)
• Ultrasound initial diagnostic study of choice
 Ultrasound shows stones, wall thickening, duct dilatation
(not inflammation)
 HIDA has sensitivity/specificity 97% / 90%
 HIDA or PIPIDA scan is positive if GB is not visualized =
cystic duct obstruction, best test for cholecystitis

Immediate surgical consult
Air in biliary tree, fever, jaundice,
diabetic, elderly, immuno-compromised
1/16/2007

UNSOM: EMR
Gallbladder Ultrasound

1/16/2007

UNSOM: EMR
Pancreatitis (1)
• Causes
Alcohol or gallstones the most common
Drugs: thiazides / estrogens / salicylates /
acetaminophen / antibiotics [ metronidazole,
sulfonamides, erythromycin, nitrofurantoin]
Metabolic disorders [hyperlipidemias,
hypercalcemia, DKA, uremia]
Viral infections [mumps, hepatitis, mono, many
others]
Bacterial infections [salmonella, streptococcus,
mycoplasma, legionella, many others]
1/16/2007

UNSOM: EMR
Pancreatitis (2)
• Ranson’s criteria (prognostic)
• On admission
Age > 55

Glucose > 200 mg
/ dl
WBC > 16,000
SGOT > 250
LDH > 350

• At 48 hours
Decreased in HCT >
10%
Increase in BUN > 5 mg
/ dl
Ca++ below 8 mg / dl
pAO2 < 60 mmHg
Base deficit > 4 mEq / L
Rapid fluid sequestration
(over 6L)

3 positives = severe disease
1/16/2007

UNSOM: EMR
Pancreatitis (3)
• Amylase
Multiple non-pancreatic sources
Height of amylase not necessarily related to
severity

• Lipase
May be more sensitive than amylase
More specific than amylase
Closely follows clinical course

• Plain x-ray

1/16/2007

Colon cutoff = dilation only over pancreas
Pancreatic calcification
Sentinel loop = small bowel air over pancreas
Imaging study of choice - contrast CT

UNSOM: EMR
Sentinel Loop (Pancreatitis)

1/16/2007

UNSOM: EMR
Pancreatitis complications
•
•
•
•
•

1/16/2007

Pseudocyst, necrosis
Hyperglycemia, hypocalcemia
Volume loss, acidosis, GI bleed
ARDS, DIC, renal failure
Death

UNSOM: EMR
GI Bleeding
Definitions
• Hematemesis - UGI proximal to ligament of
Treitz
• Hematochezia
Maroon stools
Very rapid UGI bleed (uncommon)
Usually colon or small bowel bleed

• Melena - black tarry stools - usually UGI bleed,
color from effects of acid and digestion on
blood (GI protein breakdown of blood causes
increased BUN)
1/16/2007

UNSOM: EMR
Upper GI Bleeding Sites
• A prior site of GI bleeding is often not the site
of subsequent bleeds (best example = variceal
bleed, half of subsequent bleeds are from
another site)
• UGI sites
MCC PUD (45-50%) usually duodenal
Gastritis (15-30%) (alcohol, NSAIDS)
Varices (10-15%) 1/3 of UGI bleed deaths
Mallory - Weiss esophageal tears (5-10%)
Esophagitis (5-10%) (MCC in pregnancy
Duodenitis (less than 5%)
1/16/2007

UNSOM: EMR
Upper GI Bleeding
Risk Factors for Increased Mortality
•
•
•
•
•
•
•

1/16/2007

Advancing age
SBP < 100 + hr > 100
Hematochezia
Varices
Jaundice
Hemoglobin < 10 g/dl
Co-morbid conditions

UNSOM: EMR
• A 67-year-old woman presents after three episodes of
hematemesis. She denies significant past medical history
and is taking only an over-the-counter medication for
osteoarthritis. She appears anxious and diaphoretic.
During the interview, she vomits 250 mL of bright red
blood. Physical examination is notable for blood pressure
79/58, pulse 122, moderate epigastric abdominal
tenderness and bloody stool. Which of the following is
most likely to control the bleeding?
• A. Bedside esophagogastroduodenoscopy
• B. Nasogastric tube placement with lavage
• C. Omeprazole infusion followed by vasopressin drip
• D. Sengstaken-Blakemore tube
1/16/2007

UNSOM: EMR
• A 67-year-old woman presents after three episodes of
hematemesis. She denies significant past medical history
and is taking only an over-the-counter medication for
osteoarthritis. She appears anxious and diaphoretic.
During the interview, she vomits 250 mL of bright red
blood. Physical examination is notable for blood pressure
79/58, pulse 122, moderate epigastric abdominal
tenderness and bloody stool. Which of the following is
most likely to control the bleeding?
• A. Bedside esophagogastroduodenoscopy
• B. Nasogastric tube placement with lavage
• C. Omeprazole infusion followed by vasopressin drip
• D. Sengstaken-Blakemore tube
1/16/2007

UNSOM: EMR
UGIB Management
• PPI (No benefit?)
• Octreotide for variceal bleed, decreases
splanchnic flow (No benefit?)
• Vasopressin for variceal if delay to endoscopy
• Only clear benefit from antibiotics in cirrhotics
• Sengstaken-Blakemore/Minnesota tube last
resort for esophageal varices

1/16/2007

UNSOM: EMR
Lower GI Bleeding (1)
Sites

• MCC Upper GI bleed
• Diverticulosis
• Angiodysplasia (AV malformations), associated
with HTN and aortic stenosis - usually right colon
• Aortoenteric fistula, esp if previous AAA repair
Erosion of synthetic vascular graft into gut
(often preceded by premonitory bleed)
• Cancer / polyps, IBD, rectal disease
• Hemorrhoids: MCC of rectal bleeding
• Anal fissure – MCC of minor LGI bleeding in infants
to age 5
1/16/2007

UNSOM: EMR
Low risk LGIB – send home?
•
•
•
•
•
•
•

1/16/2007

No comorbid disease
Normal vitals
Negative or trace positive stool guiac
Negative NG lavage (if performed)
Normal H/H
Good support/reliable
24 hour follow up

UNSOM: EMR
Osler-Weber-Rendu Syndrome
• Autosomal dominant vascular anomaly
• Multiple small telangiectases of the skin,
mucous membranes, GI tract
• Recurrent episodes of GI bleeding, gross and
occult

1/16/2007

UNSOM: EMR
Pediatric GI Bleeding (1)
Under 2 Months
• Upper
Bleeding diathesis
Swallowed maternal blood
Vascular malformation
• Lower
MCC is Meckel’s diverticulum (50%)
Congenital GI duplications
Intussusception
Necrotizing enterocolitis
Swallowed maternal blood
Vascular malformation
Volvulus
1/16/2007

UNSOM: EMR
Pediatric GI Bleeding (2)
Necrotizing Enterocolitis
• Predisposed
Premature neonates
Hypoxia
Hypothermia
Polycythemia
Umbilical catheters
• Mucosal edema to full thickness necrosis
• Distention, tenderness, dehydration, vomiting
• X-ray - ileus, bowel wall thickening, pneumatosis
intestinalis (late)
1/16/2007

UNSOM: EMR
Neonatal Necrotizing Enterocolitis

1/16/2007

Med-Challenger • EM
UNSOM: EMR
Pediatric GI Bleeding (3)
Under 2 Years
• Upper
Bleeding diathesis
Foreign body
Gastroenteritis
Traumatic hemobilia
Vascular
malformation
Mallory-Weiss tear

1/16/2007

• Lower
Anal fissure
Congenital dupl.
Gastroenteritis
HUS
HS purpura
Inflammatory bowel
disease
Intussusception
Meckel’s
diverticulum
Milk allergy
Polyps
UNSOM: EMR
Pediatric GI Bleeding (4)
Lower GI Bleeding Sites (1)

• Meckel’s diverticulum
 Congenital anomaly, 2% of population
 Typically diagnosed age < 2
 Located 40 cm from ileocecal jnx, free or attached to
umbilicus
 Ectopic production of gastric acid (30-50%)
 Peptic ulceration causes bleed
 Most common cause of significant LGI bleeding in
children
 Can mimic appy, may initiate intussusception, or
volvulus
Painless “bright red” bleeding
(most common clinical presentation)
1/16/2007

UNSOM: EMR
• A 11-month-old boy is brought in by his mother
after she noticed a large amount of dark red blood
in his diaper. He appears well and has normal
vital signs and a benign abdominal examination.
Rectal examination is remarkable for blood
without an obvious source. Which of the following
is needed to confirm the suspected diagnosis?
• A. Abdominal ultrasound examination
• B. Additional history on diet
• C. Apt test
• D. Nuclear medicine scan
1/16/2007

UNSOM: EMR
• A 11-month-old boy is brought in by his mother
after she noticed a large amount of dark red blood
in his diaper. He appears well and has normal
vital signs and a benign abdominal examination.
Rectal examination is remarkable for blood
without an obvious source. Which of the following
is needed to confirm the suspected diagnosis?
• A. Abdominal ultrasound examination
• B. Additional history on diet
• C. Apt test
• D. Nuclear medicine scan
1/16/2007

UNSOM: EMR
Pediatric GI Bleeding (5)
Lower GI Bleeding Sites (2)

• Intussusception

Sudden, intermittent pain, vertical sausage
mass in 50%
“Currant jelly” stool
Second most common cause of lower GI
bleeding in children
Most common cause of bowel obstruction in
first 2 yrs.
BE = diagnostic and therapeutic

1/16/2007

Lead points
Adults = polyp, cancer
Child = Meckel’s, lymphoid patch

UNSOM: EMR
Hernias (1)
• Inguinal - most common
Direct - does not
involve passage
through the inguinal
canal
Indirect - involves
inguinal canal (most
common)
• Femoral – femoral
canal, usually female,
below the inguinal
ligament, strangulation /
incarceration common
1/16/2007

UNSOM: EMR
Hernias (2)

• Umbilical
Congenital: newborns - blacks > whites; females >
males, strangulation / incarceration rare
Acquired: women, obesity, pregnancy & ascites,
strangulation / incarceration common
• Pantaloon : Indirect + direct at same time
• Spigelian (lateral ventral): level of arcuate line lateral
to rectus abdominus, difficult to diagnose, CT / US
• Richter - incarceration of a single wall of a
hollow viscus
• Incarcerated = irreducible (highest incidence of
inguinal incarceration = 1st year)
• Strangulated = irreducible with vascular compromise
(don’t manually reduce)
1/16/2007

UNSOM: EMR
Ileus
• Ileus = cessation of normal peristalsis without
mechanical obstruction
• Continuous pain, distention, decreased bowel
sounds, minimal or no tenderness, no flatus or
BM, usually self limiting
• Ileus is more common than mechanical bowel
obstruction
• X-rays show entire bowel with dilated, fluidfilled loops
1/16/2007

UNSOM: EMR
Bowel Obstruction
• Small bowel
(1) adhesions, (2) hernias, (3) malignancy
Generally more intense pain and more vomiting
and less distention than large bowel obstruction
X-ray - “step ladder” plicae circulares - traverse
bowel width
• Large bowel
(1) cancer, (2) diverticulitis, (3) sigmoid volvulus
X-ray: haustral pattern (doesn’t traverse entire
bowel width)
• “Closed-loop” obstruction dangerous = perforation
Can occur in colon if ileocecal valve is
competent
1/16/2007

UNSOM: EMR
Small Bowel Obstruction

14
1/16/2007

UNSOM: EMR
Small Bowel Obstruction

1/16/2007

UNSOM: EMR
1/16/2007

UNSOM: EMR
Volvulus

• Sigmoid volvulus
Elderly, debilitated
•
Chronic motility
disorder
Insidious onset, most
recur
X-ray: inverted u, loops
project obliquely to
RUQ
 Sigmoidoscopy may
be therapeutic
Third most common cause of
large bowel obstruction behind
(diverticular, tumor)
1/16/2007

Cecal (15 -20%)
Young (35 -55), runner
Congenital freely
mobile cecum
Acute onset
X-ray: kidney shaped
loop, LUQ,
Requires surgery
The most common cause in
pregnancy
UNSOM: EMR
Sigmoid Volvulus

1/16/2007

UNSOM: EMR
Cecal Volvulus

1/16/2007

17

UNSOM: EMR
Bowel Perforation

• Large bowel > small bowel
• Mechanism: inflammation, ulceration, trauma,
obstruction
• Causes - diverticular disease (the most common
cause), appendicitis (especially at extremes of
age), colitis / IBD, ischemia, cancer, foreign
body, PUD, radiation
• Cecum the most common site
• X-rays – may miss small amount of free air or
retroperitoneal, best view = upright chest x-ray
Ulcers are the most common cause of a visceral perforation
1/16/2007

UNSOM: EMR
Free Air; Thickened Bowel Wall

1/16/2007

UNSOM: EMR
Pediatric GI Emergencies
• Obstructive GI lesions 1st year
Gut atresia
Inguinal hernia
Malrotation, +/- volvulus
Volvulus around congenital band
Intussusception
Meconium ileus (associated with CF)
Hirschsprung’s disease
Duplication cysts of intestine

BE is diagnostic study
of choice after plain x-ray
1/16/2007

UNSOM: EMR
Pediatric GI Emergencies
Obstructive Newborn GI Lesions 1st Year

• Intussusception
MCC surgical abdomen/obstruction 3mo – 6yr
Ileocolic most common (85%)
Peak incidence - age 5 to 9 months / most occur
before 2
Classic triad only in 30% (colicky pain, vomiting,
currant jelly stool)
Paroxysms of colicky pain is the most specific
symptom
KUB: “coiled spring”
Infants less than one can have profound
listlessness as well
Children with Henoch-Schönlein purpura are at
increased risk
Ultrasound can be diagnostic as well as BE
1/16/2007

UNSOM: EMR
Intussusception

1/16/2007

UNSOM: EMR
Intussusception - Barium Enema

1/16/2007

15

UNSOM: EMR
Intussusception - Barium Enema

1/16/2007

UNSOM: EMR
Pediatric GI Emergencies
Obstructive Newborn GI lesions 1st year

• Malrotation +/- volvulus
First year of life > first month
Early diagnosis is crucial to prevent gangrene
of midgut
Abnormal rotation & fixation
X-ray: loop of bowel over-riding the liver is
suggestive (double bubble)
Acute abdomen, shock, rigid / distended
abdomen, bilious vomiting
Bilious vomiting / signs of obstruction = prompt
surgical consultation
1/16/2007

UNSOM: EMR
Pediatric GI Emergencies
Obstructive Newborn GI Lesions 1st Year
• Pyloric stenosis
Non-bilious projectile vomiting
Hypochloremic metabolic alkalosis
First born males, familial propensity 50%
Third week to third month of life
Palpable “olive”: mass lateral margin right
rectus muscle at liver edge
Ultrasound (20%) false negative
UGI: delayed gastric emptying, string sign

1/16/2007

UNSOM: EMR
• What is the most common cause of small
bowel obstruction in children?
• A. Adhesions
• B. Hernia
• C. Intussusception
• D. Midgut volvulus

1/16/2007

UNSOM: EMR
• What is the most common cause of small
bowel obstruction in children?
• A. Adhesions
• B. Hernia
• C. Intussusception
• D. Midgut volvulus

1/16/2007

UNSOM: EMR
Constipation

• Most common digestive complaint in United States,
2.5 million visits
• 30-40% > 65 years old
• Acute causes: obstruction, medication (narcotics, Ca2+
blockers, psych. meds, Fe, antacids)
• Common cause: fiber + fluid intake + exercise
• Chronic causes: slow growing tumor, thyroid,
parathyroid, lead, neurologic dysfunction
• Rectal exam for: fecal impaction, rectal mass, heme +
stool, anal fissure
• Treatment: diet/behavior changes, medical adjuncts,
underlying cause
1/16/2007

MUST RULE OUT OBSTRUCTION

UNSOM: EMR
Inflammatory Bowel Disease
•
•
•
•
•

Crohn’s disease & ulcerative colitis
Idiopathic, chronic
High rate of colon CA with disease > 10 years
Exacerbation / remission pattern
Bimodal age distribution peaks between 20’s
and 60’s
• Extracutaneous manifestations - arthritis
(20%), dermatologic (4%), hepatobiliary (4%),
vascular (1.3%) - also uveitis
• Tx: sulfasalazine, mesalamine, prednisone,
metronidazole, ciprofloxacin
1/16/2007

UNSOM: EMR
Regional Enteritis - Crohn’s Disease
• Chronic inflammatory disease of the entire GI
tract

• Segmental involvement is characteristic =
“skip lesions”
• Abdominal pain, cramps, diarrhea (sometimes
bloody), fever, perianal fissures, fistulas or
abscesses or rectal prolapse (90%), toxic
megacolon
• Gross blood uncommon
• ↑ oxalate absorption of terminal ilium leads to
nephrolithiasis
1/16/2007

UNSOM: EMR
Ulcerative Colitis
• Chronic inflammatory disease - colon
• Similar GI symptoms to Crohn’s disease
Major finding = bloody diarrhea
Toxic megacolon
Gross distention (over 8 cm)
Transverse colon
Systemic toxicity
Peritonitis
• Rectum, small bowel not affect (unlike Crohn’s)
• Colon cancer = 10 - 30 times greater risk
1/16/2007

UNSOM: EMR
Mesenteric Ischemia
• Risk factors - dysrhythmias (a. fib), low flow &
hypercoagulable states, vascular disease
• Deadly / generally elderly / early angiography
• Causes:
 Embolic *(30%)
 Arterial thrombus *(10%)
 Venous thrombus (10%)
 Nonocclusive (50%)

*Sudden onset with
pain out of proportion
to physical findings

• Leukocytosis (present in most cases), acidosis,
hyperphosphatemia, hyperamylasemia - all
inconsistently present
• Avoid digoxin, beta-blockers, vasopressors
(decrease splanchnic blood-flow)
1/16/2007

UNSOM: EMR
Mesenteric Ischemia Imaging
•
•
•
•

Thickened bowel wall
Pneumointestinalis (air in bowel wall)
Air in portal vein
“Thumb printing” = submucosal hemorrhage
All infrequently seen

Mainstay of diagnosis = arteriography

1/16/2007

UNSOM: EMR
Appendicitis (1)
• Luminal obstruction

inflammation

infection

• Anorexia often present
•
•
•
•
•
•
•

Increased perforation in elderly and small children
Pain migrating periumbilical to RLQ is specific
Late pregnancy - moves lateral and superior
BE - mass effect and non-filling
KUB - appendicolith (1%)
Ultrasound - dilated, non-compressible >6mm
Spiral CT – usually diagnostic
Most common cause of surgical abdomen

1/16/2007

UNSOM: EMR
Appendicolith

1/16/2007

19

UNSOM: EMR
Appendicitis (2)
• Confounders = situs inversus, retrocecal, pregnancy
malrotation, very long appendix
 Result-uncommon pain location: right upper quadrant,
back, flank, testicular, suprapubic
• Rovsing’s sign = LLQ palpation
RLQ pain
Psoas sign = RLQ pain on thigh extension while lying in
left lateral decubitus position
Obturator sign = RLQ pain with internal rotation of the
flexed right thigh
• Most common symptom: anorexia, nausea and vomiting
• R sided tenderness most common sign
• Rebound, rectal and referred tenderness common
• Psoas/obturator sign uncommon
1/16/2007

UNSOM: EMR
Diverticular disease

1/16/2007

UNSOM: EMR
Diverticulitis (1)
• Pain is the most common symptom
Steady, deep, LLQ

• Bowel habits may be altered - diarrhea or
constipation
• May mimic appendicitis if copious redundant
sigmoid colon
• Intraluminal pressure is greatest in the sigmoid
(most diverticula there)

1/16/2007

UNSOM: EMR
Diverticulitis (2)
• Manifestations = pain (inflammation / infection)
and bleeding; pain left side, bleeding right side
• Free perforation is rare / most are contained to
the mesentery
• May cause urinary frequency / urgency due to
irritation of underlying GU structures
• Colon cancer may be in the differential

• Tx: fiber, abx (Cipro/Metro), analgesics
1/16/2007

UNSOM: EMR
Diarrhea
• Viral
 Most common cause of diarrhea 50-70% of cases
 Mostly winter / spring / children / day care
 Rotavirus, adenovirus calicivirus, enterovirus, Norwalk agent
“RACE to Norwalk”
 Rotavirus MCC pediatric cause of diarrhea 50%
 Self-limiting / fecal-oral / community outbreak

1/16/2007

UNSOM: EMR
Diarrhea - Invasive
• Invades mucosa
inflammation (stool WBCs)
and bleeding (degree varies by pathogen),
 fever, rash, arthritis, septicemia

• E. coli 0157:HS

 Hamburger, petting zoo, raw milk, untreated water
 Can cause HUS (children) and TTP (elderly)
 No ABX recommended may increase risk of HUS

1/16/2007

UNSOM: EMR
Diarrhea - Invasive (2)
• Shigella
Very infectious,
high fever, febrile
seizures, watery bloody

Most common
cause of bloody
diarrhea

1/16/2007

• Salmonella
 Very common bacterial diarrhea
(U.S.)
 Watery / mucoid
 Pet turtles, amphibians, eggs,
chickens
 Osteomyelitis can occur in sicklers
(autosplenectomy) and those with
splenectomy

Systemic toxicity =
typhoid fever
(low WBC and relative carrier state
 Antibiotics increase bradycardia,
abdominal septic)
(give if sick /pain, no diarrhea)

UNSOM: EMR
Diarrhea - Invasive (3)
• Campylobacter
Most common cause of bacterial diarrhea
Hard to culture / water-borne (raw milk)
Invasive enterotoxin
60-70% with bloody diarrhea (gross or occult)
Erythromycin (children), fluoroquinolone (adults)
Acute infection associated with development of
Guillain-Barré syndrome

• Vibrio
Parahaemolyticus - oysters, clams, crabs,
2 -12 hour latency
Vulnificus - oysters, shellfish increased morbidity /
mortality with pre-existent liver disease
1/16/2007

UNSOM: EMR
Diarrhea - Invasive (4)
• Yersinia enterocolitica
Invasive gram pos bacteria
Increasing evidence, most common in
childhood
Can mimic appendicitis
Fever
Colicky abdominal pain (may be prolonged)
Diarrhea
May be persist 10-14 days

• Diagnosis: fecal WBC stain positive, stool
C&S
• Treatment: uncomplicated - supportive only
complicated - TMP-SMX, quinolones

1/16/2007

UNSOM: EMR
Diarrhea - Protozoan (1)
• Giardia
Most common US intestinal parasite

Beavers, deer, stream contamination
Stools floating, frothy, foul-smelling, flatulence
Multiple stool specimens may be needed to
identify cysts and / or trophozoites
Metronidazole
Homosexuals, campers, pregnancy

1/16/2007

UNSOM: EMR
Diarrhea - Protozoan (2)
• Amebiasis (entamoeba histolyticus)
Spread between family members and sexual
partners
Fecal / oral - anal intercourse
Diarrhea can be bloody
Extra-intestinal manifestations (5%)
Liver abscess most common (“chocolate cysts”)
Pericarditis, pleuropulm disease, cerebral amebiasis

Wide variety of presentations
Asymptomatic cyst passer
Colitis
Cerebral amebiasis
1/16/2007

UNSOM: EMR
Diarrhea Protozoan (3)
• Cryptosporidium
Intestinal protozoan parasites
MCC of chronic diarrhea in AIDS
Contaminated water supply; recent outbreaks
Children, animal handlers; immunocompromised
Ingestion of oocysts; trophozoites attack intestinal
membrane
1 week incubation, severe watery diarrhea,
abdominal pain

• Diagnosis: Oocyst in stool
• Treatment: Fluid replacement, CDC rec’s
nitazoxanide, or parmomycin plus azithro
1/16/2007

UNSOM: EMR
Diarrhea - Toxigenic (1)
•
•
•
•
•

1/16/2007

Bacteria producing enterotoxin
Food-borne
Diarrhea: watery, voluminous
Minor fever, no septicemia
No WBC or RBC in stool

UNSOM: EMR
Diarrhea - Toxigenic (2)
• Staph (toxin)
Contaminated foods
GI overgrowth from antibiotics
Ham, poultry, dairy products, potato salad
MCC of food-borne disease
Symptoms within 6 hours of ingestion
Usually afebrile, no abx

• E. coli

Water contaminated by feces
MCC Traveler’s diarrhea
No readily available diagnostic tests
TMP / SMX, cipro

1/16/2007

UNSOM: EMR
Diarrhea - Toxigenic (3)

• Clostridium perfringens
Common, large outbreaks
Meat and poultry source
Enterotoxin mediated
6 hours (longer onset)
Watery diarrhea
Severe abdominal cramps
Fecal WBC / RBC negative
Treatment: fluids; no abx
• Vibrio – cholera
Copious watery diarrhea= “rice water stools”
Severe fluid & electrolyte problems
Treatment: fluids, ciprofloxacin, TMP-SMX
1/16/2007

UNSOM: EMR
Diarrhea – Toxigenic
• Bacillus Cereus
• Aerobic spore forming pod
• Common in rice, especially Chinese
restaurants
• Spores germinate when boiled rice is not
refrigerated
• Two forms:
 Emetic: 2 – 3 hours post ingestion (much like Staph)
 Diarrheal: 6 – 14 hours (much like Clostridia)

• Also from vegetables and meat
• Self limited; no specific therapy or test
1/16/2007

UNSOM: EMR
Diarrhea - Toxigenic (4)
• Scombroid poisoning
Named for fish (suborder) = tuna, mackerel,
mahimahi (most frequent cause), related species
Heat - stable toxin from bacterial action on dark meat fish
Histamine - like toxin / rapid symptom onset (30
min)
Fish - tastes “peppery”
Facial flushing, diarrhea, throbbing headache,
abdominal cramps, palpitations
Give antihistamines and H2 blockers
Suspect when multiple patients have “allergic
reaction”
1/16/2007

UNSOM: EMR
Diarrhea - Toxigenic (5)
• Ciguatera
S.E. US, tropical and subtropical waters
Grouper, snapper, king fish
Fish eat certain dinoflagellates in spring /
summer, that contain toxins harmful to those
eating the fish
Muscle weakness, paresthesias (perioral,
burning hand / feet), distorted or reversed
temperature sensation, vomiting, diarrhea
Neuro symptoms worsened with alcohol
No specific treatment, symptoms can be
permanent
1/16/2007

UNSOM: EMR
Pseudomembranous Enterocolitis
•
•
•
•
•
•
•
•
•

1/16/2007

Varieties = neonatal, postop, antibiotic-related
Due to overgrowth of toxin-producing C. difficile
Begins 7 - 10 days after beginning antibiotics
Patients may be quite sick - fever, toxic, profuse
diarrhea, dehydration
Diagnosis via immunoassay for toxin
Inflammatory disease, membrane - like yellow
plaques
Treatment by stopping precipitating antibiotics
Treat with metronidazole or vancomycin orally
No anti-diarrheals

UNSOM: EMR
Botulism

• Characteristics
Heat-labile neurotoxin, short onset (half hour)
Inadequately processed canned foods
Bulbar symptoms / descending paralysis /
anticholinergic findings
• Infantile
Floppy baby, constipation, feeble cry
Honey can be source
Most common in breast-fed / also less severe in
this subset
• Adult
Diplopia (most common early finding), dysphonia,
ptosis, dysarthria, dysphagia
Anticholinergic symptoms - urinary retention, pupil
abnormalities, dry mouth, abd. cramps, nausea
and vomiting

1/16/2007

UNSOM: EMR
Rectal Prolapse
• Full thickness protrusion of rectum through anal
canal
• Sensation of rectal mass
• In children, intussusception more likely
• Differentiation from internal hemorrhoids &
intussusception
Intussusception – can place finger between
protruding rectum and anus
Internal hemorrhoids – fold of mucosa radiates
out like spoke on a wheel
Rectal prolapse – folds of mucosa circular
1/16/2007

UNSOM: EMR
Rectal Prolapse

1/16/2007

UNSOM: EMR
Hemorrhoids
• Engorgement, prolapse, or thrombosis of the
hemorrhoid veins
• Internal located at 2, 5, 9 o'clock position
• Risk factors: constipation, pregnancy, ascites, portal
hypertension
• Painless ,self limited, BRBPR,common presentation
• Treatment
Non complicated (nonsurgical): sitz bath, laxatives,
topical steroids, fiber
Complicated: large, incarcerated, strangulated,
intractable pain require surgery
Thrombosed: elliptical incision to remove clot
1/16/2007

UNSOM: EMR
Anal Fissure

• Most common causes of painful rectal bleeding in
adults and children
• 90% posterior midline
• Non-midline fissures should suggest more serious
conditions
IBD, CA, sexual abuse
• Sharp cutting pain, especially with bowel movement,
blood-streaked stool
• Perianal hygiene, sitz baths
Fistula in Ano
Tract between rectum and skin
Causes drainage and itching
Consider Crohn’s Disease
1/16/2007

UNSOM: EMR
Perianal Fissure

1/16/2007

UNSOM: EMR
Rectal Trauma
• Causes:
Penetrating 80%
Blunt 10%
Iatrogenic
Foreign body
• Must consider GU & colon injuries
• Rectal foreign body
60% removed in ED
High-riding or sharp require general anesthesia
Sigmoidoscopy after removal
1/16/2007

UNSOM: EMR
Rectal Foreign Body

1/16/2007

UNSOM: EMR
GI Miscellaneous (1)
• BE and colonoscopy are relatively
contraindicated in diverticulitis (fear of
perforation)
• Hypoglycemia in alcoholics may not respond to
glucagon because liver glycogen stores are
depleted
• AIDS patients with diarrhea usually have stool
specimens positive for pathogens; due to the
numerous causes, empiric therapy is not
advised
1/16/2007

UNSOM: EMR
GI Miscellaneous (2)
• Extension of a perirectal abscess = ischiorectal
abscess
• Prolapsed, irreducible internal hemorrhoids
require urgent surgery

• In most alcoholics with low-grade amylase
elevations, the source is non-pancreatic
• Most common serious complication of a
Sengstaken - Blakemore tube = aspiration /
suffocation
1/16/2007

UNSOM: EMR
1/16/2007

UNSOM: EMR

More Related Content

What's hot

Pancreatitisnew1 141115142107-conversion-gate01-перетворено
Pancreatitisnew1 141115142107-conversion-gate01-перетвореноPancreatitisnew1 141115142107-conversion-gate01-перетворено
Pancreatitisnew1 141115142107-conversion-gate01-перетвореноssuser347386
 
Gastroenterology
GastroenterologyGastroenterology
GastroenterologyBen Lesold
 
Liver abscess , case presentation
Liver abscess , case presentation  Liver abscess , case presentation
Liver abscess , case presentation Anupam Ghimire
 
Acute pancreatitis
Acute pancreatitisAcute pancreatitis
Acute pancreatitisAnkit Pandey
 
Obstructive Jaundice and Anesthesia
Obstructive Jaundice and AnesthesiaObstructive Jaundice and Anesthesia
Obstructive Jaundice and AnesthesiaDr.S.N.Bhagirath ..
 
Acute pancreatitis
Acute pancreatitis Acute pancreatitis
Acute pancreatitis Mohsin Khan
 
Pancreatitis
PancreatitisPancreatitis
PancreatitisMeha
 
Clinical Cases in Gastroenterology and Hepatology
Clinical Cases in Gastroenterology and HepatologyClinical Cases in Gastroenterology and Hepatology
Clinical Cases in Gastroenterology and HepatologyAhmed Adel
 
Nursing assessment and Management clients with Pancreatic disorders
Nursing assessment and Management clients with Pancreatic disordersNursing assessment and Management clients with Pancreatic disorders
Nursing assessment and Management clients with Pancreatic disordersANILKUMAR BR
 
Pancreatitis
PancreatitisPancreatitis
PancreatitisLm Huq
 

What's hot (20)

pancreatitis
pancreatitispancreatitis
pancreatitis
 
Pancreatitisnew1 141115142107-conversion-gate01-перетворено
Pancreatitisnew1 141115142107-conversion-gate01-перетвореноPancreatitisnew1 141115142107-conversion-gate01-перетворено
Pancreatitisnew1 141115142107-conversion-gate01-перетворено
 
Pancreatitis
PancreatitisPancreatitis
Pancreatitis
 
Gastroenterology
GastroenterologyGastroenterology
Gastroenterology
 
Gastroenterology
GastroenterologyGastroenterology
Gastroenterology
 
Liver abscess , case presentation
Liver abscess , case presentation  Liver abscess , case presentation
Liver abscess , case presentation
 
Uremia
UremiaUremia
Uremia
 
Pancreatitis
PancreatitisPancreatitis
Pancreatitis
 
Biliary pancreatitis
Biliary pancreatitisBiliary pancreatitis
Biliary pancreatitis
 
Acute pancreatitis
Acute pancreatitisAcute pancreatitis
Acute pancreatitis
 
Acute pancreatitis
Acute pancreatitisAcute pancreatitis
Acute pancreatitis
 
Obstructive Jaundice and Anesthesia
Obstructive Jaundice and AnesthesiaObstructive Jaundice and Anesthesia
Obstructive Jaundice and Anesthesia
 
Gastroenterology
GastroenterologyGastroenterology
Gastroenterology
 
Chronic pancreatitis
Chronic pancreatitisChronic pancreatitis
Chronic pancreatitis
 
Acute pancreatitis
Acute pancreatitis Acute pancreatitis
Acute pancreatitis
 
Pancreatitis
PancreatitisPancreatitis
Pancreatitis
 
Clinical Cases in Gastroenterology and Hepatology
Clinical Cases in Gastroenterology and HepatologyClinical Cases in Gastroenterology and Hepatology
Clinical Cases in Gastroenterology and Hepatology
 
Nursing assessment and Management clients with Pancreatic disorders
Nursing assessment and Management clients with Pancreatic disordersNursing assessment and Management clients with Pancreatic disorders
Nursing assessment and Management clients with Pancreatic disorders
 
Acute pancreatitis
Acute pancreatitisAcute pancreatitis
Acute pancreatitis
 
Pancreatitis
PancreatitisPancreatitis
Pancreatitis
 

Viewers also liked

Acute Pancreatitis (According to American College of Gastroenterology 2013 gu...
Acute Pancreatitis (According to American College of Gastroenterology 2013 gu...Acute Pancreatitis (According to American College of Gastroenterology 2013 gu...
Acute Pancreatitis (According to American College of Gastroenterology 2013 gu...Jibran Mohsin
 
Gi bleed peds awais
Gi bleed peds awaisGi bleed peds awais
Gi bleed peds awaisAli Shazir
 
Penetrat neck-injury-100331
Penetrat neck-injury-100331Penetrat neck-injury-100331
Penetrat neck-injury-100331Anahi Kielbasa
 
GEMC - Gastrointestinal Bleeding in the Pediatric Patient
GEMC - Gastrointestinal Bleeding in the Pediatric PatientGEMC - Gastrointestinal Bleeding in the Pediatric Patient
GEMC - Gastrointestinal Bleeding in the Pediatric PatientOpen.Michigan
 
Radiology of digestive system
Radiology of digestive systemRadiology of digestive system
Radiology of digestive systemghalan
 
Basic chest x ray interpretation
Basic chest x ray interpretationBasic chest x ray interpretation
Basic chest x ray interpretationHiba Ashibany
 
22 acute pancreatitis
22 acute pancreatitis22 acute pancreatitis
22 acute pancreatitisinternalmed
 

Viewers also liked (14)

Acute Pancreatitis (According to American College of Gastroenterology 2013 gu...
Acute Pancreatitis (According to American College of Gastroenterology 2013 gu...Acute Pancreatitis (According to American College of Gastroenterology 2013 gu...
Acute Pancreatitis (According to American College of Gastroenterology 2013 gu...
 
Gi bleed peds awais
Gi bleed peds awaisGi bleed peds awais
Gi bleed peds awais
 
ACES: Rabies / Tick Illness
ACES: Rabies / Tick IllnessACES: Rabies / Tick Illness
ACES: Rabies / Tick Illness
 
ACES: CV Drugs / Pesticides
ACES: CV Drugs / PesticidesACES: CV Drugs / Pesticides
ACES: CV Drugs / Pesticides
 
Penetrat neck-injury-100331
Penetrat neck-injury-100331Penetrat neck-injury-100331
Penetrat neck-injury-100331
 
Esophageal disease
Esophageal diseaseEsophageal disease
Esophageal disease
 
Mellss surgery y3 intestinal obstruction
Mellss surgery y3 intestinal obstructionMellss surgery y3 intestinal obstruction
Mellss surgery y3 intestinal obstruction
 
Acute pancreatitis SP
Acute pancreatitis SPAcute pancreatitis SP
Acute pancreatitis SP
 
ACES: Hand
ACES: HandACES: Hand
ACES: Hand
 
GEMC - Gastrointestinal Bleeding in the Pediatric Patient
GEMC - Gastrointestinal Bleeding in the Pediatric PatientGEMC - Gastrointestinal Bleeding in the Pediatric Patient
GEMC - Gastrointestinal Bleeding in the Pediatric Patient
 
Radiology of digestive system
Radiology of digestive systemRadiology of digestive system
Radiology of digestive system
 
Chest X Ray Interpretation
Chest  X Ray InterpretationChest  X Ray Interpretation
Chest X Ray Interpretation
 
Basic chest x ray interpretation
Basic chest x ray interpretationBasic chest x ray interpretation
Basic chest x ray interpretation
 
22 acute pancreatitis
22 acute pancreatitis22 acute pancreatitis
22 acute pancreatitis
 

Similar to ITE Review: GI

Joint urology emphysematous pyelonephritis
Joint urology   emphysematous pyelonephritisJoint urology   emphysematous pyelonephritis
Joint urology emphysematous pyelonephritisAdrianne Gabucan
 
SURGICAL CONDITIONS OF THE PANCREAS.pptx
SURGICAL CONDITIONS OF THE PANCREAS.pptxSURGICAL CONDITIONS OF THE PANCREAS.pptx
SURGICAL CONDITIONS OF THE PANCREAS.pptxBiniam24
 
Clinical assessment geriatrics 1
Clinical assessment geriatrics 1Clinical assessment geriatrics 1
Clinical assessment geriatrics 1Doha Rasheedy
 
approach to Disphagia for medical students
approach to Disphagia for medical studentsapproach to Disphagia for medical students
approach to Disphagia for medical studentsYahyia Al-abri
 
2. physiology of deglutition and disorders of swallowing
2. physiology of deglutition and disorders of swallowing2. physiology of deglutition and disorders of swallowing
2. physiology of deglutition and disorders of swallowingkrishnakoirala4
 
PAINFUL HIP IN CHILDREN.pptx
PAINFUL HIP IN CHILDREN.pptxPAINFUL HIP IN CHILDREN.pptx
PAINFUL HIP IN CHILDREN.pptxpeterlikes
 
Acute vs chronic scrotal swelling
Acute vs chronic scrotal swellingAcute vs chronic scrotal swelling
Acute vs chronic scrotal swellingKochi Chia
 
Ultrasound in acute abdominal pain
Ultrasound in acute abdominal painUltrasound in acute abdominal pain
Ultrasound in acute abdominal painSuzanneCain2
 
Language development historic root
Language development historic rootLanguage development historic root
Language development historic rootMuhammad Ahmad
 
Cerebral Fat Embolism in ICU
Cerebral Fat Embolism in ICU Cerebral Fat Embolism in ICU
Cerebral Fat Embolism in ICU Dr.Tarek Sabry
 
Non-malignant Dysphagia Surgical Management
Non-malignant Dysphagia Surgical Management Non-malignant Dysphagia Surgical Management
Non-malignant Dysphagia Surgical Management jim kuok
 

Similar to ITE Review: GI (20)

Gi disorders
Gi disordersGi disorders
Gi disorders
 
Joint urology emphysematous pyelonephritis
Joint urology   emphysematous pyelonephritisJoint urology   emphysematous pyelonephritis
Joint urology emphysematous pyelonephritis
 
SURGICAL CONDITIONS OF THE PANCREAS.pptx
SURGICAL CONDITIONS OF THE PANCREAS.pptxSURGICAL CONDITIONS OF THE PANCREAS.pptx
SURGICAL CONDITIONS OF THE PANCREAS.pptx
 
Clinical assessment geriatrics 1
Clinical assessment geriatrics 1Clinical assessment geriatrics 1
Clinical assessment geriatrics 1
 
approach to Disphagia for medical students
approach to Disphagia for medical studentsapproach to Disphagia for medical students
approach to Disphagia for medical students
 
Pathology of Upper GIT
Pathology of Upper GITPathology of Upper GIT
Pathology of Upper GIT
 
2. physiology of deglutition and disorders of swallowing
2. physiology of deglutition and disorders of swallowing2. physiology of deglutition and disorders of swallowing
2. physiology of deglutition and disorders of swallowing
 
PAINFUL HIP IN CHILDREN.pptx
PAINFUL HIP IN CHILDREN.pptxPAINFUL HIP IN CHILDREN.pptx
PAINFUL HIP IN CHILDREN.pptx
 
Oncological Emergencies
Oncological EmergenciesOncological Emergencies
Oncological Emergencies
 
Acute vs chronic scrotal swelling
Acute vs chronic scrotal swellingAcute vs chronic scrotal swelling
Acute vs chronic scrotal swelling
 
Musculo skeletal system
Musculo skeletal systemMusculo skeletal system
Musculo skeletal system
 
Ultrasound in acute abdominal pain
Ultrasound in acute abdominal painUltrasound in acute abdominal pain
Ultrasound in acute abdominal pain
 
Language development historic root
Language development historic rootLanguage development historic root
Language development historic root
 
2. GIT CONDITIONS.pptx
2. GIT CONDITIONS.pptx2. GIT CONDITIONS.pptx
2. GIT CONDITIONS.pptx
 
Rare case of dysphagia - Dr Shaz Pamangadan
Rare case of dysphagia - Dr Shaz PamangadanRare case of dysphagia - Dr Shaz Pamangadan
Rare case of dysphagia - Dr Shaz Pamangadan
 
ACUTE ABDOMEN pptx
ACUTE ABDOMEN pptxACUTE ABDOMEN pptx
ACUTE ABDOMEN pptx
 
Intestinal obstruction2
Intestinal obstruction2Intestinal obstruction2
Intestinal obstruction2
 
Esophageal and extraesophageal management of GERD
Esophageal and extraesophageal management of GERDEsophageal and extraesophageal management of GERD
Esophageal and extraesophageal management of GERD
 
Cerebral Fat Embolism in ICU
Cerebral Fat Embolism in ICU Cerebral Fat Embolism in ICU
Cerebral Fat Embolism in ICU
 
Non-malignant Dysphagia Surgical Management
Non-malignant Dysphagia Surgical Management Non-malignant Dysphagia Surgical Management
Non-malignant Dysphagia Surgical Management
 

More from Las Vegas Emergency Medicine

More from Las Vegas Emergency Medicine (20)

Hyperbaric oxygen therapy elsbecker
Hyperbaric oxygen therapy   elsbeckerHyperbaric oxygen therapy   elsbecker
Hyperbaric oxygen therapy elsbecker
 
Hgb ss crisis elsbecker
Hgb ss crisis   elsbeckerHgb ss crisis   elsbecker
Hgb ss crisis elsbecker
 
Gallbladder disease galster
Gallbladder disease   galsterGallbladder disease   galster
Gallbladder disease galster
 
Em advocacy elsbecker
Em advocacy   elsbeckerEm advocacy   elsbecker
Em advocacy elsbecker
 
Chemical warfare healey
Chemical warfare   healeyChemical warfare   healey
Chemical warfare healey
 
Bioterrorism healey
Bioterrorism   healeyBioterrorism   healey
Bioterrorism healey
 
You only live twice elsbecker
You only live twice   elsbeckerYou only live twice   elsbecker
You only live twice elsbecker
 
ACES: Constipation / Thyroid-Adrenal / Elderly
ACES: Constipation / Thyroid-Adrenal / ElderlyACES: Constipation / Thyroid-Adrenal / Elderly
ACES: Constipation / Thyroid-Adrenal / Elderly
 
ACES: Ankle / Foot
ACES: Ankle / FootACES: Ankle / Foot
ACES: Ankle / Foot
 
Dave's career planning seminar
Dave's career planning seminarDave's career planning seminar
Dave's career planning seminar
 
Intralipid
IntralipidIntralipid
Intralipid
 
Blast injuries
Blast injuriesBlast injuries
Blast injuries
 
Pediatric rashes
Pediatric rashesPediatric rashes
Pediatric rashes
 
Asthma
AsthmaAsthma
Asthma
 
The history of emergency medicine
The history of emergency medicineThe history of emergency medicine
The history of emergency medicine
 
Hot As Hell
Hot As HellHot As Hell
Hot As Hell
 
Upper Respiratory Infections
Upper Respiratory InfectionsUpper Respiratory Infections
Upper Respiratory Infections
 
Toxicology Intro
Toxicology IntroToxicology Intro
Toxicology Intro
 
Skin and Soft Tissue Infections
Skin and Soft Tissue InfectionsSkin and Soft Tissue Infections
Skin and Soft Tissue Infections
 
Sinusitis and Unicorns
Sinusitis and UnicornsSinusitis and Unicorns
Sinusitis and Unicorns
 

Recently uploaded

METHODS OF ACQUIRING KNOWLEDGE IN NURSING.pptx by navdeep kaur
METHODS OF ACQUIRING KNOWLEDGE IN NURSING.pptx by navdeep kaurMETHODS OF ACQUIRING KNOWLEDGE IN NURSING.pptx by navdeep kaur
METHODS OF ACQUIRING KNOWLEDGE IN NURSING.pptx by navdeep kaurNavdeep Kaur
 
Giftedness: Understanding Everyday Neurobiology for Self-Knowledge
Giftedness: Understanding Everyday Neurobiology for Self-KnowledgeGiftedness: Understanding Everyday Neurobiology for Self-Knowledge
Giftedness: Understanding Everyday Neurobiology for Self-Knowledgeassessoriafabianodea
 
Basic principles involved in the traditional systems of medicine PDF.pdf
Basic principles involved in the traditional systems of medicine PDF.pdfBasic principles involved in the traditional systems of medicine PDF.pdf
Basic principles involved in the traditional systems of medicine PDF.pdfDivya Kanojiya
 
Presentation for Bella Mahl 2024-03-28-24-MW-Overview-Bella.pptx
Presentation for Bella Mahl 2024-03-28-24-MW-Overview-Bella.pptxPresentation for Bella Mahl 2024-03-28-24-MW-Overview-Bella.pptx
Presentation for Bella Mahl 2024-03-28-24-MW-Overview-Bella.pptxpdamico1
 
Biomechanics- Shoulder Joint!!!!!!!!!!!!
Biomechanics- Shoulder Joint!!!!!!!!!!!!Biomechanics- Shoulder Joint!!!!!!!!!!!!
Biomechanics- Shoulder Joint!!!!!!!!!!!!ibtesaam huma
 
low cost antibiotic cement nail for infected non union.pptx
low cost antibiotic cement nail for infected non union.pptxlow cost antibiotic cement nail for infected non union.pptx
low cost antibiotic cement nail for infected non union.pptxdrashraf369
 
CEHPALOSPORINS.pptx By Harshvardhan Dev Bhoomi Uttarakhand University
CEHPALOSPORINS.pptx By Harshvardhan Dev Bhoomi Uttarakhand UniversityCEHPALOSPORINS.pptx By Harshvardhan Dev Bhoomi Uttarakhand University
CEHPALOSPORINS.pptx By Harshvardhan Dev Bhoomi Uttarakhand UniversityHarshChauhan475104
 
epilepsy and status epilepticus for undergraduate.pptx
epilepsy and status epilepticus  for undergraduate.pptxepilepsy and status epilepticus  for undergraduate.pptx
epilepsy and status epilepticus for undergraduate.pptxMohamed Rizk Khodair
 
Hematology and Immunology - Leukocytes Functions
Hematology and Immunology - Leukocytes FunctionsHematology and Immunology - Leukocytes Functions
Hematology and Immunology - Leukocytes FunctionsMedicoseAcademics
 
Tans femoral Amputee : Prosthetics Knee Joints.pptx
Tans femoral Amputee : Prosthetics Knee Joints.pptxTans femoral Amputee : Prosthetics Knee Joints.pptx
Tans femoral Amputee : Prosthetics Knee Joints.pptxKezaiah S
 
Presentation on General Anesthetics pdf.
Presentation on General Anesthetics pdf.Presentation on General Anesthetics pdf.
Presentation on General Anesthetics pdf.Prerana Jadhav
 
SGK HÓA SINH NĂNG LƯỢNG SINH HỌC 2006.pdf
SGK HÓA SINH NĂNG LƯỢNG SINH HỌC 2006.pdfSGK HÓA SINH NĂNG LƯỢNG SINH HỌC 2006.pdf
SGK HÓA SINH NĂNG LƯỢNG SINH HỌC 2006.pdfHongBiThi1
 
COVID-19 (NOVEL CORONA VIRUS DISEASE PANDEMIC ).pptx
COVID-19  (NOVEL CORONA  VIRUS DISEASE PANDEMIC ).pptxCOVID-19  (NOVEL CORONA  VIRUS DISEASE PANDEMIC ).pptx
COVID-19 (NOVEL CORONA VIRUS DISEASE PANDEMIC ).pptxBibekananda shah
 
Wessex Health Partners Wessex Integrated Care, Population Health, Research & ...
Wessex Health Partners Wessex Integrated Care, Population Health, Research & ...Wessex Health Partners Wessex Integrated Care, Population Health, Research & ...
Wessex Health Partners Wessex Integrated Care, Population Health, Research & ...Wessex Health Partners
 
MedDRA-A-Comprehensive-Guide-to-Standardized-Medical-Terminology.pdf
MedDRA-A-Comprehensive-Guide-to-Standardized-Medical-Terminology.pdfMedDRA-A-Comprehensive-Guide-to-Standardized-Medical-Terminology.pdf
MedDRA-A-Comprehensive-Guide-to-Standardized-Medical-Terminology.pdfSasikiranMarri
 
SWD (Short wave diathermy)- Physiotherapy.ppt
SWD (Short wave diathermy)- Physiotherapy.pptSWD (Short wave diathermy)- Physiotherapy.ppt
SWD (Short wave diathermy)- Physiotherapy.pptMumux Mirani
 
Introduction to Sports Injuries by- Dr. Anjali Rai
Introduction to Sports Injuries by- Dr. Anjali RaiIntroduction to Sports Injuries by- Dr. Anjali Rai
Introduction to Sports Injuries by- Dr. Anjali RaiGoogle
 
PERFECT BUT PAINFUL TKR -ROLE OF SYNOVECTOMY.pptx
PERFECT BUT PAINFUL TKR -ROLE OF SYNOVECTOMY.pptxPERFECT BUT PAINFUL TKR -ROLE OF SYNOVECTOMY.pptx
PERFECT BUT PAINFUL TKR -ROLE OF SYNOVECTOMY.pptxdrashraf369
 
Measurement of Radiation and Dosimetric Procedure.pptx
Measurement of Radiation and Dosimetric Procedure.pptxMeasurement of Radiation and Dosimetric Procedure.pptx
Measurement of Radiation and Dosimetric Procedure.pptxDr. Dheeraj Kumar
 
Clinical Pharmacotherapy of Scabies Disease
Clinical Pharmacotherapy of Scabies DiseaseClinical Pharmacotherapy of Scabies Disease
Clinical Pharmacotherapy of Scabies DiseaseSreenivasa Reddy Thalla
 

Recently uploaded (20)

METHODS OF ACQUIRING KNOWLEDGE IN NURSING.pptx by navdeep kaur
METHODS OF ACQUIRING KNOWLEDGE IN NURSING.pptx by navdeep kaurMETHODS OF ACQUIRING KNOWLEDGE IN NURSING.pptx by navdeep kaur
METHODS OF ACQUIRING KNOWLEDGE IN NURSING.pptx by navdeep kaur
 
Giftedness: Understanding Everyday Neurobiology for Self-Knowledge
Giftedness: Understanding Everyday Neurobiology for Self-KnowledgeGiftedness: Understanding Everyday Neurobiology for Self-Knowledge
Giftedness: Understanding Everyday Neurobiology for Self-Knowledge
 
Basic principles involved in the traditional systems of medicine PDF.pdf
Basic principles involved in the traditional systems of medicine PDF.pdfBasic principles involved in the traditional systems of medicine PDF.pdf
Basic principles involved in the traditional systems of medicine PDF.pdf
 
Presentation for Bella Mahl 2024-03-28-24-MW-Overview-Bella.pptx
Presentation for Bella Mahl 2024-03-28-24-MW-Overview-Bella.pptxPresentation for Bella Mahl 2024-03-28-24-MW-Overview-Bella.pptx
Presentation for Bella Mahl 2024-03-28-24-MW-Overview-Bella.pptx
 
Biomechanics- Shoulder Joint!!!!!!!!!!!!
Biomechanics- Shoulder Joint!!!!!!!!!!!!Biomechanics- Shoulder Joint!!!!!!!!!!!!
Biomechanics- Shoulder Joint!!!!!!!!!!!!
 
low cost antibiotic cement nail for infected non union.pptx
low cost antibiotic cement nail for infected non union.pptxlow cost antibiotic cement nail for infected non union.pptx
low cost antibiotic cement nail for infected non union.pptx
 
CEHPALOSPORINS.pptx By Harshvardhan Dev Bhoomi Uttarakhand University
CEHPALOSPORINS.pptx By Harshvardhan Dev Bhoomi Uttarakhand UniversityCEHPALOSPORINS.pptx By Harshvardhan Dev Bhoomi Uttarakhand University
CEHPALOSPORINS.pptx By Harshvardhan Dev Bhoomi Uttarakhand University
 
epilepsy and status epilepticus for undergraduate.pptx
epilepsy and status epilepticus  for undergraduate.pptxepilepsy and status epilepticus  for undergraduate.pptx
epilepsy and status epilepticus for undergraduate.pptx
 
Hematology and Immunology - Leukocytes Functions
Hematology and Immunology - Leukocytes FunctionsHematology and Immunology - Leukocytes Functions
Hematology and Immunology - Leukocytes Functions
 
Tans femoral Amputee : Prosthetics Knee Joints.pptx
Tans femoral Amputee : Prosthetics Knee Joints.pptxTans femoral Amputee : Prosthetics Knee Joints.pptx
Tans femoral Amputee : Prosthetics Knee Joints.pptx
 
Presentation on General Anesthetics pdf.
Presentation on General Anesthetics pdf.Presentation on General Anesthetics pdf.
Presentation on General Anesthetics pdf.
 
SGK HÓA SINH NĂNG LƯỢNG SINH HỌC 2006.pdf
SGK HÓA SINH NĂNG LƯỢNG SINH HỌC 2006.pdfSGK HÓA SINH NĂNG LƯỢNG SINH HỌC 2006.pdf
SGK HÓA SINH NĂNG LƯỢNG SINH HỌC 2006.pdf
 
COVID-19 (NOVEL CORONA VIRUS DISEASE PANDEMIC ).pptx
COVID-19  (NOVEL CORONA  VIRUS DISEASE PANDEMIC ).pptxCOVID-19  (NOVEL CORONA  VIRUS DISEASE PANDEMIC ).pptx
COVID-19 (NOVEL CORONA VIRUS DISEASE PANDEMIC ).pptx
 
Wessex Health Partners Wessex Integrated Care, Population Health, Research & ...
Wessex Health Partners Wessex Integrated Care, Population Health, Research & ...Wessex Health Partners Wessex Integrated Care, Population Health, Research & ...
Wessex Health Partners Wessex Integrated Care, Population Health, Research & ...
 
MedDRA-A-Comprehensive-Guide-to-Standardized-Medical-Terminology.pdf
MedDRA-A-Comprehensive-Guide-to-Standardized-Medical-Terminology.pdfMedDRA-A-Comprehensive-Guide-to-Standardized-Medical-Terminology.pdf
MedDRA-A-Comprehensive-Guide-to-Standardized-Medical-Terminology.pdf
 
SWD (Short wave diathermy)- Physiotherapy.ppt
SWD (Short wave diathermy)- Physiotherapy.pptSWD (Short wave diathermy)- Physiotherapy.ppt
SWD (Short wave diathermy)- Physiotherapy.ppt
 
Introduction to Sports Injuries by- Dr. Anjali Rai
Introduction to Sports Injuries by- Dr. Anjali RaiIntroduction to Sports Injuries by- Dr. Anjali Rai
Introduction to Sports Injuries by- Dr. Anjali Rai
 
PERFECT BUT PAINFUL TKR -ROLE OF SYNOVECTOMY.pptx
PERFECT BUT PAINFUL TKR -ROLE OF SYNOVECTOMY.pptxPERFECT BUT PAINFUL TKR -ROLE OF SYNOVECTOMY.pptx
PERFECT BUT PAINFUL TKR -ROLE OF SYNOVECTOMY.pptx
 
Measurement of Radiation and Dosimetric Procedure.pptx
Measurement of Radiation and Dosimetric Procedure.pptxMeasurement of Radiation and Dosimetric Procedure.pptx
Measurement of Radiation and Dosimetric Procedure.pptx
 
Clinical Pharmacotherapy of Scabies Disease
Clinical Pharmacotherapy of Scabies DiseaseClinical Pharmacotherapy of Scabies Disease
Clinical Pharmacotherapy of Scabies Disease
 

ITE Review: GI

  • 2. Dysphagia (1) • • • • Difficulty swallowing Solids: mechanical/obstructive Solids/liquids: motility disorder Oropharyngeal dysphagia (transfer): neuromuscular disorder (CVA) • Progressive (CA) vs. non-progressive (web) • Strictures 2° reflux (can mimic CA) • Work up  Esophagram  Endoscopy  Esophageal motility studies 1/16/2007 UNSOM: EMR
  • 3. Dysphagia (2) • Infectious: Botulism, diptheria, polio, rabies, Sydenham’s chorea (rheumatic fever), tetanus • Immunologic: Scleroderma, multiple sclerosis, myasthenia, ALS, polymyositis, amyloidosis • Motor dysfunction: CN palsy (posterior CVA), diabetic neuropathy Achalasia (vomit undigested food) Aperistalsis of esophagus (loss of Auerbach’s plexus in the esophagus) Tx: CCB (Diltiazem, nifedipine) –Botox, dilation, myomotomy 1/16/2007 UNSOM: EMR
  • 4. • A 32-year-old woman presents with chest pain that has worsened over the past 2 months. She says it gets worse when she lies flat or exercises and after she eats or drinks quickly. She has no significant past medical history, but her husband says she has lost about 10 pounds recently and has been throwing up undigested food. What are the expected diagnostic findings? • A. Diffuse ST-segment elevation and PR-interval depression • B. Dilated esophagus proximal to a beaklike lower esophageal sphincter • C. Gastric inflammatory changes • D. White matter plaques in the brainstem 1/16/2007 UNSOM: EMR
  • 5. • A 32-year-old woman presents with chest pain that has worsened over the past 2 months. She says it gets worse when she lies flat or exercises and after she eats or drinks quickly. She has no significant past medical history, but her husband says she has lost about 10 pounds recently and has been throwing up undigested food. What are the expected diagnostic findings? • A. Diffuse ST-segment elevation and PR-interval depression • B. Dilated esophagus proximal to a beaklike lower esophageal sphincter • C. Gastric inflammatory changes • D. White matter plaques in the brainstem 1/16/2007 UNSOM: EMR
  • 6. Dysphagia (3) Mechanical • Zenker’s diverticulum Pharyngoesophageal pouch Proximal: above the UES Elderly, regurgitation of undigested food • Cancer: MCC = squamous Risk factors: smoking, achalasia, caustic ingestion • Extraluminal obstruction / tumor 1/16/2007 UNSOM: EMR
  • 7. Dysphagia (4) Mechanical • Strictures GERD, chronic inflammation, occur in distal esophagus • Schatzki’s ring Fibrous structure distal esophagus MCC of intermittent dysphagia, steakhouse syndrome • Webs (occurs intermittently) Circumferential mucosal outpouchings Congenital or acquired Plummer - Vinson Syndrome = symptomatic hypopharyngeal webs + iron deficiency anemia 1/16/2007 UNSOM: EMR
  • 8. Odynophagia • Odynophagia - pain upon swallowing spasm - painful muscle contraction • Causes of esophagitis Reflux Infection: candida, herpes, immunosuppression: (HIV, DM, steroid use, CA) Inflammatory conditions: infection, radiation, trauma, foreign body • Admit dysphagia, odynophagia Bleeding Rupture Severe dehydration Malnutrition 1/16/2007 UNSOM: EMR
  • 9. • Which of the following patients requires oral fluconazole treatment? • A. 17-year-old girl with both dysphagia and odynophagia refractory to acid suppression therapy who also has multiple allergies • B. 27-year-old man with chest pain and severe odynophagia who also has asthma and is HIV positive • C. 47-year-old man with transport dysphagia for solids initially and now liquids who also smokes • D. 55-year-old man with halitosis, transfer dysphagia, and neck fullness 1/16/2007 UNSOM: EMR
  • 10. • Which of the following patients requires oral fluconazole treatment? • A. 17-year-old girl with both dysphagia and odynophagia refractory to acid suppression therapy who also has multiple allergies • B. 27-year-old man with chest pain and severe odynophagia who also has asthma and is HIV positive • C. 47-year-old man with transport dysphagia for solids initially and now liquids who also smokes • D. 55-year-old man with halitosis, transfer dysphagia, and neck fullness 1/16/2007 UNSOM: EMR
  • 11. Hiccups (Singultus) • Involuntary stimulation of the respiratory reflex with spastic contraction of inspiratory muscles on closed glottis • Benign causes: gastric distention, smoking, ETOH, change is environmental temperature • Persistent: damage to vagus/phrenic nerve/CNS  Continue with sleep: organic  Relieved with sleep: psychogenic • Organic  CNS: neoplasm, MS, ICP  PUD, tonsillitis, goiter, pericarditis, pacemaker, STEMI 1/16/2007 UNSOM: EMR
  • 12. Esophageal Rupture (1) • MCC iatrogenic #1: Endoscopy #2: Dilation MCC in ED: NG tube Diagnosis by esophagram • Mallory - Weiss - partial thickness tear Location: GE junction 5-15 % of UGI bleeds Vomiting, retching Risk factors: ETOH, hiatal hernia Spontaneous resolution common 1/16/2007 UNSOM: EMR
  • 13. Esophageal Rupture (2) • Boerhaave’s Syndrome - full thickness tear Males usually, age 40-60 Typically associated with alcohol (50%) Typically left posterior distal rupture Chemical, then infectious mediastinitis Severe chest pain, shock, sepsis Air in mediastinum (Hamman’s crunch) Pyopneumothorax Gastrografin (water soluble) UGI Fluids, Antibiotics, Surgical consult • X-ray: mediastinal air, left pleural effusion, pneumothorax, widened mediastinum 1/16/2007 UNSOM: EMR
  • 15. Esophageal Foreign Bodies • Levels of narrowing MCC: Cricopharyngeus muscle (C6) (<4 y/o) Aortic arch (T4) Tracheal bifurcation (T6) Gastroesophageal junction (least) (T11) • Coin x-rays AP orientation = trachea (same plane as vocal cord orientation) Transverse orientation = esophagus 1/16/2007 UNSOM: EMR
  • 18. Esophageal Foreign Bodies (3) • 10-20% require some intervention • 1% demand surgical treatment • Most foreign bodies will pass if they traverse the pylorus • Soft drink pull tabs - may not show up on x-ray 1/16/2007 UNSOM: EMR
  • 19. Esophageal Foreign Bodies (4) • Indications for endoscopy Sharp / elongated Button batteries Perforation Nickel / quarter at C6 (pediatric) In esophagus > 24 hours 1/16/2007 UNSOM: EMR
  • 20. Esophageal Foreign Bodies (5) • Button batteries  Double density radiographically  Must always be removed from esophagus immediately  Rapid burns with perforation < 6 hours (Lithium worse)  Batteries do not need to be removed: Passed esophagus, asymptomatic Passed the pylorus <48 hours  Most will pass completely in 48-72 hours, serial radiography • Treatment: broad-spectrum ABX, surgical consultation 1/16/2007 UNSOM: EMR
  • 21. Foreign Bodies (6) Sharp objects > 5cm long & 2cm wide Magnet + metal All others: serial exam / x-rays Fish/Chicken bones or plastic  CT 1/16/2007 UNSOM: EMR
  • 22. Sharp Foreign Body Sharp Foreign Body 1/16/2007 UNSOM: EMR
  • 24. Esophageal Food Impaction • Most patients with food impaction have underlying esophageal pathology • Must evaluate for cause after dislodgement • Treatment options: Glucagon - relaxes distal esophageal sphincter Nifedipine - reduces lower esophageal tone Carbonated beverages - gaseous distention may push the bolus into the stomach Endoscopy No papain (meat tenderizer) 1/16/2007 UNSOM: EMR
  • 25. Caustic Ingestions (1) • Acids (+/- bad) Coagulation necrosis No ongoing tissue necrosis • Alkali (bad) Liquefaction necrosis (pH 12.5) Ongoing tissue necrosis • Severity Nature, volume and concentration Tissue contact time Presence or absence of stomach contents Tonicity of pyloric sphincter 1/16/2007 5 UNSOM: EMR
  • 26. Caustic Ingestions (2) • Inconsistent relationship between oral signs / symptoms and esophageal findings • All patients with 2-3° burns are symptomatic • Diluents - water / milk only for solid alkali • No neutralizers = exothermic generation of heat Endoscopy best diagnostic tool • Complications Early: acute airway compromise due to edema, perforation Late: stricture, perforation 1/16/2007 UNSOM: EMR
  • 27. Peptic Ulcer Disease • Incidence decreasing in general population and increasing in the elderly (liberal use of NSAIDs) • MCC Duodenal (80%), gastric (20%) • Helicobacter pylori responsible for most • Predisposing factors: • Treatment: - antibiotics against H. pylori (amox, clarithro, metro) - histamine blockers (histamines stimulate acid inhibitors) - parietal cell inhibitors (omeprazole) - ulcer surface protectants (sucralfate) • Complications: 1/16/2007 - smoking, alcohol - type O blood - NSAIDs and steroids - bleeding - perforation (can cause pancreatitis) (do upright CXR for free air) - obstruction UNSOM: EMR
  • 28. Bilirubin (1) • Breakdown product of hemoglobin • Hyperbilirubinemia • Unconjugated (> 85%) (“indirect reacting”) Increased bilirubin load (hemolysis) Inability to conjugate (Gilbert’s, neonatal, sepsis) • Conjugated (< 30%) (“direct reacting”) Decreased ability to excrete from biliary tree = cholestasis / obstruction Intrahepatic cholestasis Hepatocellular damage Damage to biliary endothelium Extrahepatic cholestasis 1/16/2007 Biliary outflow obstruction (stones, mass, congenital inflammation, CHF) UNSOM: EMR
  • 29. Bilirubin (2) • Conjugated bilirubin in bowel is converted by gut bacteria to urobilinogen • Urobilinogen is absorbed from the gut into the circulation and excreted in urine • If jaundice is present but urine urobilinogen is negative = excess unconjugated hyperbilirubinemia • If jaundice is present but excess positive urine urobilinogen = excess conjugated bilirubin 1/16/2007 UNSOM: EMR
  • 30. Hepatitis (1) • Causes - viral and toxic Malaise, jaundice, increased SGOT, increased bilirubin Alcoholic hepatitis Abnormal protime is a marker indicating significant liver dysfunction if elevated, consider altering or holding doses of livermetabolized drugs • Viral Type A 1/16/2007 Fecal - oral, onset 2 weeks post-exposure Prophylaxis - immune globulin within 2 weeks of exposure (travelers, household contacts) UNSOM: EMR
  • 31. Hepatitis (2) • Viral Type B Percutaneous, parenteral or sexual exposure Onset 1-6 mo (mean = 75 days) post-exposure Complications = cirrhosis, liver cancer, carrier state (10%) • Markers HBsAg: HBsAb: HBcAb: HBeAg: + early (before enzymes increase) Infective + 2-6 mo after clearance of HBsAg Immune + 2 wks after + HBsAg * persists for life + implies high infectivity *May be the only positive marker during the window when HBsAg declining and HBsAb increasing 1/16/2007 UNSOM: EMR
  • 32. Hepatitis (3) • Hepatitis B exposure - source known HBsAg positive • Unvaccinated HBIG ASAP + vaccination (0, 1 mo, 6 mo) • Vaccinated Incomplete series- vaccine booster Known responder- test for HBsAb if > 10, no rx; if < 10 HBIG and vaccine booster Known non - responder - HBIG x 2 (0, 30 days) 1/16/2007 UNSOM: EMR
  • 33. Hepatitis (4) • Hepatitis B exposure - source unknown • Unvaccinated Initiate vaccination • Vaccinated Same as for HBsAg positive source • HBIG only recommended if source or situation maybe high risk for exposure 1/16/2007 UNSOM: EMR
  • 34. Hepatitis (5) • Viral Type C Percutaneous, parenteral or sexual exposure Usual cause of non-A, non-B hepatitis High carrier rate, higher incidence in HIV Cirrhosis / liver cancer (50%) 2% seroconversion • Indications for hospitalization (any hepatitis) Encephalopathy, PT/INR significantly increased, dehydration, hypoglycemia, bilirubin over 20, age over 45, immunosuppression, diagnosis uncertain 1/16/2007 UNSOM: EMR
  • 35. Hepatic Encephalopathy • Precipitants = “LIVER” (Librium [sedatives], Infection, Volume loss, Electrolytes disorders, Red blood cells in the gut [a major cause]) • Others: dietary protein excess, worsening hepatocellular function • Early sign = “sleep inversion” - sleeping during the day / awake at night • Asterixis (“liver flap”) • Ammonia levels: arterial more helpful than venous • Check for hypoglycemia!!! • Treatment: Oral or rectal neomycin / lactulose / decrease dietary protein / avoid sedatives / avoid bicarbonate (alkalosis can worsen encephalopathy) 1/16/2007 UNSOM: EMR
  • 36. Spontaneous Bacterial Peritonitis • Occurs with chronic liver disease Portal hypertension bowel edema migration and leakage of enteric organisms (E. coli 50%, enterococcus 25%) • Abdominal tenderness, worsening ascites, encephalopathy, fever, sepsis, shock • Diagnosis: paracentesis with increased WBC PMN >250/ul • Tx: Ceftriaxone, ppx: Cipro or Bactrim 1/16/2007 UNSOM: EMR
  • 37. • A 57-year-old man with a history of cirrhosis presents with acute renal failure. He denies recent illness and is not taking any nephrotoxic medications. He is well hydrated; his urinalysis is negative. Which of the following is the definitive treatment? • A. Hydration • B. Liver transplant • C. Renal transplant • D. Transjugular intrahepatic portosystemic shunt 1/16/2007 UNSOM: EMR
  • 38. • A 57-year-old man with a history of cirrhosis presents with acute renal failure. He denies recent illness and is not taking any nephrotoxic medications. He is well hydrated; his urinalysis is negative. Which of the following is the definitive treatment? • A. Hydration • B. Liver transplant • C. Renal transplant • D. Transjugular intrahepatic portosystemic shunt 1/16/2007 UNSOM: EMR
  • 39. Gallbladder (1) • Stones = mostly bilirubin / cholesterol (radiolucent) • Biliary colic = pain, vomiting, due to obstruction by stones without inflammation • Cholecystitis (stone-related = calculous) MCC of abdominal pain in the elderly OR Obstruction distention pain / vomiting / inflammation infection (usually E. coli, Klebsiella) increased WBCs • Rupture of stone into small bowel with obstruction at ileocecal valve = GALLSTONE ILEUS Air in biliary tree (from bowel) = pneumobilia 1/16/2007 UNSOM: EMR
  • 40. Gallbladder (2) • Acalculous cholecystitis No stones 5-10% of cases Usually a complication of another process (trauma, burn, postpartum, postop, narcotics) Patients often quite sick Likely cause of GB perforation Increased risk with diabetes and elderly Greater morbidity than calculous variety • Ascending cholangitis Infection spreading through biliary tree Charcot’s triad = jaundice, fever, RUQ pain 1/16/2007 UNSOM: EMR
  • 42. Gallbladder (3) • Ultrasound initial diagnostic study of choice  Ultrasound shows stones, wall thickening, duct dilatation (not inflammation)  HIDA has sensitivity/specificity 97% / 90%  HIDA or PIPIDA scan is positive if GB is not visualized = cystic duct obstruction, best test for cholecystitis Immediate surgical consult Air in biliary tree, fever, jaundice, diabetic, elderly, immuno-compromised 1/16/2007 UNSOM: EMR
  • 44. Pancreatitis (1) • Causes Alcohol or gallstones the most common Drugs: thiazides / estrogens / salicylates / acetaminophen / antibiotics [ metronidazole, sulfonamides, erythromycin, nitrofurantoin] Metabolic disorders [hyperlipidemias, hypercalcemia, DKA, uremia] Viral infections [mumps, hepatitis, mono, many others] Bacterial infections [salmonella, streptococcus, mycoplasma, legionella, many others] 1/16/2007 UNSOM: EMR
  • 45. Pancreatitis (2) • Ranson’s criteria (prognostic) • On admission Age > 55 Glucose > 200 mg / dl WBC > 16,000 SGOT > 250 LDH > 350 • At 48 hours Decreased in HCT > 10% Increase in BUN > 5 mg / dl Ca++ below 8 mg / dl pAO2 < 60 mmHg Base deficit > 4 mEq / L Rapid fluid sequestration (over 6L) 3 positives = severe disease 1/16/2007 UNSOM: EMR
  • 46. Pancreatitis (3) • Amylase Multiple non-pancreatic sources Height of amylase not necessarily related to severity • Lipase May be more sensitive than amylase More specific than amylase Closely follows clinical course • Plain x-ray 1/16/2007 Colon cutoff = dilation only over pancreas Pancreatic calcification Sentinel loop = small bowel air over pancreas Imaging study of choice - contrast CT UNSOM: EMR
  • 48. Pancreatitis complications • • • • • 1/16/2007 Pseudocyst, necrosis Hyperglycemia, hypocalcemia Volume loss, acidosis, GI bleed ARDS, DIC, renal failure Death UNSOM: EMR
  • 49. GI Bleeding Definitions • Hematemesis - UGI proximal to ligament of Treitz • Hematochezia Maroon stools Very rapid UGI bleed (uncommon) Usually colon or small bowel bleed • Melena - black tarry stools - usually UGI bleed, color from effects of acid and digestion on blood (GI protein breakdown of blood causes increased BUN) 1/16/2007 UNSOM: EMR
  • 50. Upper GI Bleeding Sites • A prior site of GI bleeding is often not the site of subsequent bleeds (best example = variceal bleed, half of subsequent bleeds are from another site) • UGI sites MCC PUD (45-50%) usually duodenal Gastritis (15-30%) (alcohol, NSAIDS) Varices (10-15%) 1/3 of UGI bleed deaths Mallory - Weiss esophageal tears (5-10%) Esophagitis (5-10%) (MCC in pregnancy Duodenitis (less than 5%) 1/16/2007 UNSOM: EMR
  • 51. Upper GI Bleeding Risk Factors for Increased Mortality • • • • • • • 1/16/2007 Advancing age SBP < 100 + hr > 100 Hematochezia Varices Jaundice Hemoglobin < 10 g/dl Co-morbid conditions UNSOM: EMR
  • 52. • A 67-year-old woman presents after three episodes of hematemesis. She denies significant past medical history and is taking only an over-the-counter medication for osteoarthritis. She appears anxious and diaphoretic. During the interview, she vomits 250 mL of bright red blood. Physical examination is notable for blood pressure 79/58, pulse 122, moderate epigastric abdominal tenderness and bloody stool. Which of the following is most likely to control the bleeding? • A. Bedside esophagogastroduodenoscopy • B. Nasogastric tube placement with lavage • C. Omeprazole infusion followed by vasopressin drip • D. Sengstaken-Blakemore tube 1/16/2007 UNSOM: EMR
  • 53. • A 67-year-old woman presents after three episodes of hematemesis. She denies significant past medical history and is taking only an over-the-counter medication for osteoarthritis. She appears anxious and diaphoretic. During the interview, she vomits 250 mL of bright red blood. Physical examination is notable for blood pressure 79/58, pulse 122, moderate epigastric abdominal tenderness and bloody stool. Which of the following is most likely to control the bleeding? • A. Bedside esophagogastroduodenoscopy • B. Nasogastric tube placement with lavage • C. Omeprazole infusion followed by vasopressin drip • D. Sengstaken-Blakemore tube 1/16/2007 UNSOM: EMR
  • 54. UGIB Management • PPI (No benefit?) • Octreotide for variceal bleed, decreases splanchnic flow (No benefit?) • Vasopressin for variceal if delay to endoscopy • Only clear benefit from antibiotics in cirrhotics • Sengstaken-Blakemore/Minnesota tube last resort for esophageal varices 1/16/2007 UNSOM: EMR
  • 55. Lower GI Bleeding (1) Sites • MCC Upper GI bleed • Diverticulosis • Angiodysplasia (AV malformations), associated with HTN and aortic stenosis - usually right colon • Aortoenteric fistula, esp if previous AAA repair Erosion of synthetic vascular graft into gut (often preceded by premonitory bleed) • Cancer / polyps, IBD, rectal disease • Hemorrhoids: MCC of rectal bleeding • Anal fissure – MCC of minor LGI bleeding in infants to age 5 1/16/2007 UNSOM: EMR
  • 56. Low risk LGIB – send home? • • • • • • • 1/16/2007 No comorbid disease Normal vitals Negative or trace positive stool guiac Negative NG lavage (if performed) Normal H/H Good support/reliable 24 hour follow up UNSOM: EMR
  • 57. Osler-Weber-Rendu Syndrome • Autosomal dominant vascular anomaly • Multiple small telangiectases of the skin, mucous membranes, GI tract • Recurrent episodes of GI bleeding, gross and occult 1/16/2007 UNSOM: EMR
  • 58. Pediatric GI Bleeding (1) Under 2 Months • Upper Bleeding diathesis Swallowed maternal blood Vascular malformation • Lower MCC is Meckel’s diverticulum (50%) Congenital GI duplications Intussusception Necrotizing enterocolitis Swallowed maternal blood Vascular malformation Volvulus 1/16/2007 UNSOM: EMR
  • 59. Pediatric GI Bleeding (2) Necrotizing Enterocolitis • Predisposed Premature neonates Hypoxia Hypothermia Polycythemia Umbilical catheters • Mucosal edema to full thickness necrosis • Distention, tenderness, dehydration, vomiting • X-ray - ileus, bowel wall thickening, pneumatosis intestinalis (late) 1/16/2007 UNSOM: EMR
  • 61. Pediatric GI Bleeding (3) Under 2 Years • Upper Bleeding diathesis Foreign body Gastroenteritis Traumatic hemobilia Vascular malformation Mallory-Weiss tear 1/16/2007 • Lower Anal fissure Congenital dupl. Gastroenteritis HUS HS purpura Inflammatory bowel disease Intussusception Meckel’s diverticulum Milk allergy Polyps UNSOM: EMR
  • 62. Pediatric GI Bleeding (4) Lower GI Bleeding Sites (1) • Meckel’s diverticulum  Congenital anomaly, 2% of population  Typically diagnosed age < 2  Located 40 cm from ileocecal jnx, free or attached to umbilicus  Ectopic production of gastric acid (30-50%)  Peptic ulceration causes bleed  Most common cause of significant LGI bleeding in children  Can mimic appy, may initiate intussusception, or volvulus Painless “bright red” bleeding (most common clinical presentation) 1/16/2007 UNSOM: EMR
  • 63. • A 11-month-old boy is brought in by his mother after she noticed a large amount of dark red blood in his diaper. He appears well and has normal vital signs and a benign abdominal examination. Rectal examination is remarkable for blood without an obvious source. Which of the following is needed to confirm the suspected diagnosis? • A. Abdominal ultrasound examination • B. Additional history on diet • C. Apt test • D. Nuclear medicine scan 1/16/2007 UNSOM: EMR
  • 64. • A 11-month-old boy is brought in by his mother after she noticed a large amount of dark red blood in his diaper. He appears well and has normal vital signs and a benign abdominal examination. Rectal examination is remarkable for blood without an obvious source. Which of the following is needed to confirm the suspected diagnosis? • A. Abdominal ultrasound examination • B. Additional history on diet • C. Apt test • D. Nuclear medicine scan 1/16/2007 UNSOM: EMR
  • 65. Pediatric GI Bleeding (5) Lower GI Bleeding Sites (2) • Intussusception Sudden, intermittent pain, vertical sausage mass in 50% “Currant jelly” stool Second most common cause of lower GI bleeding in children Most common cause of bowel obstruction in first 2 yrs. BE = diagnostic and therapeutic 1/16/2007 Lead points Adults = polyp, cancer Child = Meckel’s, lymphoid patch UNSOM: EMR
  • 66. Hernias (1) • Inguinal - most common Direct - does not involve passage through the inguinal canal Indirect - involves inguinal canal (most common) • Femoral – femoral canal, usually female, below the inguinal ligament, strangulation / incarceration common 1/16/2007 UNSOM: EMR
  • 67. Hernias (2) • Umbilical Congenital: newborns - blacks > whites; females > males, strangulation / incarceration rare Acquired: women, obesity, pregnancy & ascites, strangulation / incarceration common • Pantaloon : Indirect + direct at same time • Spigelian (lateral ventral): level of arcuate line lateral to rectus abdominus, difficult to diagnose, CT / US • Richter - incarceration of a single wall of a hollow viscus • Incarcerated = irreducible (highest incidence of inguinal incarceration = 1st year) • Strangulated = irreducible with vascular compromise (don’t manually reduce) 1/16/2007 UNSOM: EMR
  • 68. Ileus • Ileus = cessation of normal peristalsis without mechanical obstruction • Continuous pain, distention, decreased bowel sounds, minimal or no tenderness, no flatus or BM, usually self limiting • Ileus is more common than mechanical bowel obstruction • X-rays show entire bowel with dilated, fluidfilled loops 1/16/2007 UNSOM: EMR
  • 69. Bowel Obstruction • Small bowel (1) adhesions, (2) hernias, (3) malignancy Generally more intense pain and more vomiting and less distention than large bowel obstruction X-ray - “step ladder” plicae circulares - traverse bowel width • Large bowel (1) cancer, (2) diverticulitis, (3) sigmoid volvulus X-ray: haustral pattern (doesn’t traverse entire bowel width) • “Closed-loop” obstruction dangerous = perforation Can occur in colon if ileocecal valve is competent 1/16/2007 UNSOM: EMR
  • 73. Volvulus • Sigmoid volvulus Elderly, debilitated • Chronic motility disorder Insidious onset, most recur X-ray: inverted u, loops project obliquely to RUQ  Sigmoidoscopy may be therapeutic Third most common cause of large bowel obstruction behind (diverticular, tumor) 1/16/2007 Cecal (15 -20%) Young (35 -55), runner Congenital freely mobile cecum Acute onset X-ray: kidney shaped loop, LUQ, Requires surgery The most common cause in pregnancy UNSOM: EMR
  • 76. Bowel Perforation • Large bowel > small bowel • Mechanism: inflammation, ulceration, trauma, obstruction • Causes - diverticular disease (the most common cause), appendicitis (especially at extremes of age), colitis / IBD, ischemia, cancer, foreign body, PUD, radiation • Cecum the most common site • X-rays – may miss small amount of free air or retroperitoneal, best view = upright chest x-ray Ulcers are the most common cause of a visceral perforation 1/16/2007 UNSOM: EMR
  • 77. Free Air; Thickened Bowel Wall 1/16/2007 UNSOM: EMR
  • 78. Pediatric GI Emergencies • Obstructive GI lesions 1st year Gut atresia Inguinal hernia Malrotation, +/- volvulus Volvulus around congenital band Intussusception Meconium ileus (associated with CF) Hirschsprung’s disease Duplication cysts of intestine BE is diagnostic study of choice after plain x-ray 1/16/2007 UNSOM: EMR
  • 79. Pediatric GI Emergencies Obstructive Newborn GI Lesions 1st Year • Intussusception MCC surgical abdomen/obstruction 3mo – 6yr Ileocolic most common (85%) Peak incidence - age 5 to 9 months / most occur before 2 Classic triad only in 30% (colicky pain, vomiting, currant jelly stool) Paroxysms of colicky pain is the most specific symptom KUB: “coiled spring” Infants less than one can have profound listlessness as well Children with Henoch-Schönlein purpura are at increased risk Ultrasound can be diagnostic as well as BE 1/16/2007 UNSOM: EMR
  • 81. Intussusception - Barium Enema 1/16/2007 15 UNSOM: EMR
  • 82. Intussusception - Barium Enema 1/16/2007 UNSOM: EMR
  • 83. Pediatric GI Emergencies Obstructive Newborn GI lesions 1st year • Malrotation +/- volvulus First year of life > first month Early diagnosis is crucial to prevent gangrene of midgut Abnormal rotation & fixation X-ray: loop of bowel over-riding the liver is suggestive (double bubble) Acute abdomen, shock, rigid / distended abdomen, bilious vomiting Bilious vomiting / signs of obstruction = prompt surgical consultation 1/16/2007 UNSOM: EMR
  • 84. Pediatric GI Emergencies Obstructive Newborn GI Lesions 1st Year • Pyloric stenosis Non-bilious projectile vomiting Hypochloremic metabolic alkalosis First born males, familial propensity 50% Third week to third month of life Palpable “olive”: mass lateral margin right rectus muscle at liver edge Ultrasound (20%) false negative UGI: delayed gastric emptying, string sign 1/16/2007 UNSOM: EMR
  • 85. • What is the most common cause of small bowel obstruction in children? • A. Adhesions • B. Hernia • C. Intussusception • D. Midgut volvulus 1/16/2007 UNSOM: EMR
  • 86. • What is the most common cause of small bowel obstruction in children? • A. Adhesions • B. Hernia • C. Intussusception • D. Midgut volvulus 1/16/2007 UNSOM: EMR
  • 87. Constipation • Most common digestive complaint in United States, 2.5 million visits • 30-40% > 65 years old • Acute causes: obstruction, medication (narcotics, Ca2+ blockers, psych. meds, Fe, antacids) • Common cause: fiber + fluid intake + exercise • Chronic causes: slow growing tumor, thyroid, parathyroid, lead, neurologic dysfunction • Rectal exam for: fecal impaction, rectal mass, heme + stool, anal fissure • Treatment: diet/behavior changes, medical adjuncts, underlying cause 1/16/2007 MUST RULE OUT OBSTRUCTION UNSOM: EMR
  • 88. Inflammatory Bowel Disease • • • • • Crohn’s disease & ulcerative colitis Idiopathic, chronic High rate of colon CA with disease > 10 years Exacerbation / remission pattern Bimodal age distribution peaks between 20’s and 60’s • Extracutaneous manifestations - arthritis (20%), dermatologic (4%), hepatobiliary (4%), vascular (1.3%) - also uveitis • Tx: sulfasalazine, mesalamine, prednisone, metronidazole, ciprofloxacin 1/16/2007 UNSOM: EMR
  • 89. Regional Enteritis - Crohn’s Disease • Chronic inflammatory disease of the entire GI tract • Segmental involvement is characteristic = “skip lesions” • Abdominal pain, cramps, diarrhea (sometimes bloody), fever, perianal fissures, fistulas or abscesses or rectal prolapse (90%), toxic megacolon • Gross blood uncommon • ↑ oxalate absorption of terminal ilium leads to nephrolithiasis 1/16/2007 UNSOM: EMR
  • 90. Ulcerative Colitis • Chronic inflammatory disease - colon • Similar GI symptoms to Crohn’s disease Major finding = bloody diarrhea Toxic megacolon Gross distention (over 8 cm) Transverse colon Systemic toxicity Peritonitis • Rectum, small bowel not affect (unlike Crohn’s) • Colon cancer = 10 - 30 times greater risk 1/16/2007 UNSOM: EMR
  • 91. Mesenteric Ischemia • Risk factors - dysrhythmias (a. fib), low flow & hypercoagulable states, vascular disease • Deadly / generally elderly / early angiography • Causes:  Embolic *(30%)  Arterial thrombus *(10%)  Venous thrombus (10%)  Nonocclusive (50%) *Sudden onset with pain out of proportion to physical findings • Leukocytosis (present in most cases), acidosis, hyperphosphatemia, hyperamylasemia - all inconsistently present • Avoid digoxin, beta-blockers, vasopressors (decrease splanchnic blood-flow) 1/16/2007 UNSOM: EMR
  • 92. Mesenteric Ischemia Imaging • • • • Thickened bowel wall Pneumointestinalis (air in bowel wall) Air in portal vein “Thumb printing” = submucosal hemorrhage All infrequently seen Mainstay of diagnosis = arteriography 1/16/2007 UNSOM: EMR
  • 93. Appendicitis (1) • Luminal obstruction inflammation infection • Anorexia often present • • • • • • • Increased perforation in elderly and small children Pain migrating periumbilical to RLQ is specific Late pregnancy - moves lateral and superior BE - mass effect and non-filling KUB - appendicolith (1%) Ultrasound - dilated, non-compressible >6mm Spiral CT – usually diagnostic Most common cause of surgical abdomen 1/16/2007 UNSOM: EMR
  • 95. Appendicitis (2) • Confounders = situs inversus, retrocecal, pregnancy malrotation, very long appendix  Result-uncommon pain location: right upper quadrant, back, flank, testicular, suprapubic • Rovsing’s sign = LLQ palpation RLQ pain Psoas sign = RLQ pain on thigh extension while lying in left lateral decubitus position Obturator sign = RLQ pain with internal rotation of the flexed right thigh • Most common symptom: anorexia, nausea and vomiting • R sided tenderness most common sign • Rebound, rectal and referred tenderness common • Psoas/obturator sign uncommon 1/16/2007 UNSOM: EMR
  • 97. Diverticulitis (1) • Pain is the most common symptom Steady, deep, LLQ • Bowel habits may be altered - diarrhea or constipation • May mimic appendicitis if copious redundant sigmoid colon • Intraluminal pressure is greatest in the sigmoid (most diverticula there) 1/16/2007 UNSOM: EMR
  • 98. Diverticulitis (2) • Manifestations = pain (inflammation / infection) and bleeding; pain left side, bleeding right side • Free perforation is rare / most are contained to the mesentery • May cause urinary frequency / urgency due to irritation of underlying GU structures • Colon cancer may be in the differential • Tx: fiber, abx (Cipro/Metro), analgesics 1/16/2007 UNSOM: EMR
  • 99. Diarrhea • Viral  Most common cause of diarrhea 50-70% of cases  Mostly winter / spring / children / day care  Rotavirus, adenovirus calicivirus, enterovirus, Norwalk agent “RACE to Norwalk”  Rotavirus MCC pediatric cause of diarrhea 50%  Self-limiting / fecal-oral / community outbreak 1/16/2007 UNSOM: EMR
  • 100. Diarrhea - Invasive • Invades mucosa inflammation (stool WBCs) and bleeding (degree varies by pathogen),  fever, rash, arthritis, septicemia • E. coli 0157:HS  Hamburger, petting zoo, raw milk, untreated water  Can cause HUS (children) and TTP (elderly)  No ABX recommended may increase risk of HUS 1/16/2007 UNSOM: EMR
  • 101. Diarrhea - Invasive (2) • Shigella Very infectious, high fever, febrile seizures, watery bloody Most common cause of bloody diarrhea 1/16/2007 • Salmonella  Very common bacterial diarrhea (U.S.)  Watery / mucoid  Pet turtles, amphibians, eggs, chickens  Osteomyelitis can occur in sicklers (autosplenectomy) and those with splenectomy Systemic toxicity = typhoid fever (low WBC and relative carrier state  Antibiotics increase bradycardia, abdominal septic) (give if sick /pain, no diarrhea) UNSOM: EMR
  • 102. Diarrhea - Invasive (3) • Campylobacter Most common cause of bacterial diarrhea Hard to culture / water-borne (raw milk) Invasive enterotoxin 60-70% with bloody diarrhea (gross or occult) Erythromycin (children), fluoroquinolone (adults) Acute infection associated with development of Guillain-Barré syndrome • Vibrio Parahaemolyticus - oysters, clams, crabs, 2 -12 hour latency Vulnificus - oysters, shellfish increased morbidity / mortality with pre-existent liver disease 1/16/2007 UNSOM: EMR
  • 103. Diarrhea - Invasive (4) • Yersinia enterocolitica Invasive gram pos bacteria Increasing evidence, most common in childhood Can mimic appendicitis Fever Colicky abdominal pain (may be prolonged) Diarrhea May be persist 10-14 days • Diagnosis: fecal WBC stain positive, stool C&S • Treatment: uncomplicated - supportive only complicated - TMP-SMX, quinolones 1/16/2007 UNSOM: EMR
  • 104. Diarrhea - Protozoan (1) • Giardia Most common US intestinal parasite Beavers, deer, stream contamination Stools floating, frothy, foul-smelling, flatulence Multiple stool specimens may be needed to identify cysts and / or trophozoites Metronidazole Homosexuals, campers, pregnancy 1/16/2007 UNSOM: EMR
  • 105. Diarrhea - Protozoan (2) • Amebiasis (entamoeba histolyticus) Spread between family members and sexual partners Fecal / oral - anal intercourse Diarrhea can be bloody Extra-intestinal manifestations (5%) Liver abscess most common (“chocolate cysts”) Pericarditis, pleuropulm disease, cerebral amebiasis Wide variety of presentations Asymptomatic cyst passer Colitis Cerebral amebiasis 1/16/2007 UNSOM: EMR
  • 106. Diarrhea Protozoan (3) • Cryptosporidium Intestinal protozoan parasites MCC of chronic diarrhea in AIDS Contaminated water supply; recent outbreaks Children, animal handlers; immunocompromised Ingestion of oocysts; trophozoites attack intestinal membrane 1 week incubation, severe watery diarrhea, abdominal pain • Diagnosis: Oocyst in stool • Treatment: Fluid replacement, CDC rec’s nitazoxanide, or parmomycin plus azithro 1/16/2007 UNSOM: EMR
  • 107. Diarrhea - Toxigenic (1) • • • • • 1/16/2007 Bacteria producing enterotoxin Food-borne Diarrhea: watery, voluminous Minor fever, no septicemia No WBC or RBC in stool UNSOM: EMR
  • 108. Diarrhea - Toxigenic (2) • Staph (toxin) Contaminated foods GI overgrowth from antibiotics Ham, poultry, dairy products, potato salad MCC of food-borne disease Symptoms within 6 hours of ingestion Usually afebrile, no abx • E. coli Water contaminated by feces MCC Traveler’s diarrhea No readily available diagnostic tests TMP / SMX, cipro 1/16/2007 UNSOM: EMR
  • 109. Diarrhea - Toxigenic (3) • Clostridium perfringens Common, large outbreaks Meat and poultry source Enterotoxin mediated 6 hours (longer onset) Watery diarrhea Severe abdominal cramps Fecal WBC / RBC negative Treatment: fluids; no abx • Vibrio – cholera Copious watery diarrhea= “rice water stools” Severe fluid & electrolyte problems Treatment: fluids, ciprofloxacin, TMP-SMX 1/16/2007 UNSOM: EMR
  • 110. Diarrhea – Toxigenic • Bacillus Cereus • Aerobic spore forming pod • Common in rice, especially Chinese restaurants • Spores germinate when boiled rice is not refrigerated • Two forms:  Emetic: 2 – 3 hours post ingestion (much like Staph)  Diarrheal: 6 – 14 hours (much like Clostridia) • Also from vegetables and meat • Self limited; no specific therapy or test 1/16/2007 UNSOM: EMR
  • 111. Diarrhea - Toxigenic (4) • Scombroid poisoning Named for fish (suborder) = tuna, mackerel, mahimahi (most frequent cause), related species Heat - stable toxin from bacterial action on dark meat fish Histamine - like toxin / rapid symptom onset (30 min) Fish - tastes “peppery” Facial flushing, diarrhea, throbbing headache, abdominal cramps, palpitations Give antihistamines and H2 blockers Suspect when multiple patients have “allergic reaction” 1/16/2007 UNSOM: EMR
  • 112. Diarrhea - Toxigenic (5) • Ciguatera S.E. US, tropical and subtropical waters Grouper, snapper, king fish Fish eat certain dinoflagellates in spring / summer, that contain toxins harmful to those eating the fish Muscle weakness, paresthesias (perioral, burning hand / feet), distorted or reversed temperature sensation, vomiting, diarrhea Neuro symptoms worsened with alcohol No specific treatment, symptoms can be permanent 1/16/2007 UNSOM: EMR
  • 113. Pseudomembranous Enterocolitis • • • • • • • • • 1/16/2007 Varieties = neonatal, postop, antibiotic-related Due to overgrowth of toxin-producing C. difficile Begins 7 - 10 days after beginning antibiotics Patients may be quite sick - fever, toxic, profuse diarrhea, dehydration Diagnosis via immunoassay for toxin Inflammatory disease, membrane - like yellow plaques Treatment by stopping precipitating antibiotics Treat with metronidazole or vancomycin orally No anti-diarrheals UNSOM: EMR
  • 114. Botulism • Characteristics Heat-labile neurotoxin, short onset (half hour) Inadequately processed canned foods Bulbar symptoms / descending paralysis / anticholinergic findings • Infantile Floppy baby, constipation, feeble cry Honey can be source Most common in breast-fed / also less severe in this subset • Adult Diplopia (most common early finding), dysphonia, ptosis, dysarthria, dysphagia Anticholinergic symptoms - urinary retention, pupil abnormalities, dry mouth, abd. cramps, nausea and vomiting 1/16/2007 UNSOM: EMR
  • 115. Rectal Prolapse • Full thickness protrusion of rectum through anal canal • Sensation of rectal mass • In children, intussusception more likely • Differentiation from internal hemorrhoids & intussusception Intussusception – can place finger between protruding rectum and anus Internal hemorrhoids – fold of mucosa radiates out like spoke on a wheel Rectal prolapse – folds of mucosa circular 1/16/2007 UNSOM: EMR
  • 117. Hemorrhoids • Engorgement, prolapse, or thrombosis of the hemorrhoid veins • Internal located at 2, 5, 9 o'clock position • Risk factors: constipation, pregnancy, ascites, portal hypertension • Painless ,self limited, BRBPR,common presentation • Treatment Non complicated (nonsurgical): sitz bath, laxatives, topical steroids, fiber Complicated: large, incarcerated, strangulated, intractable pain require surgery Thrombosed: elliptical incision to remove clot 1/16/2007 UNSOM: EMR
  • 118. Anal Fissure • Most common causes of painful rectal bleeding in adults and children • 90% posterior midline • Non-midline fissures should suggest more serious conditions IBD, CA, sexual abuse • Sharp cutting pain, especially with bowel movement, blood-streaked stool • Perianal hygiene, sitz baths Fistula in Ano Tract between rectum and skin Causes drainage and itching Consider Crohn’s Disease 1/16/2007 UNSOM: EMR
  • 120. Rectal Trauma • Causes: Penetrating 80% Blunt 10% Iatrogenic Foreign body • Must consider GU & colon injuries • Rectal foreign body 60% removed in ED High-riding or sharp require general anesthesia Sigmoidoscopy after removal 1/16/2007 UNSOM: EMR
  • 122. GI Miscellaneous (1) • BE and colonoscopy are relatively contraindicated in diverticulitis (fear of perforation) • Hypoglycemia in alcoholics may not respond to glucagon because liver glycogen stores are depleted • AIDS patients with diarrhea usually have stool specimens positive for pathogens; due to the numerous causes, empiric therapy is not advised 1/16/2007 UNSOM: EMR
  • 123. GI Miscellaneous (2) • Extension of a perirectal abscess = ischiorectal abscess • Prolapsed, irreducible internal hemorrhoids require urgent surgery • In most alcoholics with low-grade amylase elevations, the source is non-pancreatic • Most common serious complication of a Sengstaken - Blakemore tube = aspiration / suffocation 1/16/2007 UNSOM: EMR

Editor's Notes

  1. Dysphagia vs odynophagia; Solids vs solid/liquid
  2. Different causes of dyspahgia
  3. No charcoal
  4. Early HbcAb is IgMIgG
  5. Mortality goes up to 15%
  6. Suspensory muscle of the duodenum
  7. First few days of life, late as one month
  8. Still MCC in female is inguinal
  9. LL decub
  10. Although rare after 3 yearsHSP more often ileoileal
  11. Large bowel loops
  12. Cutoff
  13. Coiled spring
  14. Full thickeness
  15. Superficial mucosal layer
  16. Lactic acidWBC 30K